[obm-l] Re: [obm-l] Dúvida e ajuda.

2022-04-10 Por tôpico Anderson Torres
Em sex., 8 de abr. de 2022 às 11:17, Pedro José  escreveu:
>
> Bom dia!
> Posso concluir que um número representado por uma infinidade de algarismos 
> decimais é racional se e somente se tem um período de repetições desses 
> algarismos?
> A ida é fácil se tiver o período é racional.
> Já a volta não sei se é verdade e se for há como provar?

Acho que dá para fazer isso mais algoritmicamente.

Um número da forma

0,(A)

onde A é um período de k dígitos (por óbvio, zeros à esquerda são
permitidos, como em 0,010101010101...) é essencialmente um racional da
forma A/..9 com k noves - ou melhor escrevendo, (A/(10^k-1)).

Já números da forma 0,B(A) onde B tem m dígitos são a mesma coisa que
10^(-m)*(B+A/(10^k-1)), o que, após simplificar, dá (maçaroca
qualquer)/(10^m*(10^k-1)).

Qualquer racional por definição é da forma p/q com q natural. Bastaria
demonstrar que todo natural q tem um múltiplo da forma
(10^m*(10^k-1)), o que sai de uma aplicação de Euler-Fermat ou mesmo
de casa do pombo.

(Aliás, quem foi o BR que traduziu "princípio do escaninho" para
"princípio de casa de pombo"?)

>
> Meu objetivo primário é saber se:
> 0,123456789112233445566778899111222333444555666777888999... é racional. As 
> reticências se referem ao aumento de mais um algarismo repetido a cada 
> sequência, ou seja a primeira aparição de 1 será 1, a 2a 11 a 3a 111 e assim 
> sucessivamente, o mesmo vale para os demais algarismos.
>
> Alguém poderia me ajudar?
> Grato,
> PJMS
>
> --
> Esta mensagem foi verificada pelo sistema de antivírus e
> acredita-se estar livre de perigo.

-- 
Esta mensagem foi verificada pelo sistema de antiv�rus e
 acredita-se estar livre de perigo.


=
Instru��es para entrar na lista, sair da lista e usar a lista em
http://www.mat.puc-rio.br/~obmlistas/obm-l.html
=


[obm-l] Re: [obm-l] Re: [obm-l] Dúvida e ajuda.

2022-04-08 Por tôpico Pedro José
Grato a todos!
Já, já tenho de voltar ao trabalho.
Depois dou uma olhada.
Mas achei a demonstração usando casa de pombos, simples e prática.
Já que tem de haver um p/q com pp temos w=x+p/q,
onde x é a parte inteira de w/q, então pq e os restos só podem q-1, uma hora tem de
repetir e aí volta a sequência.
Mas saindo do trabalho dou uma olhada.
Mais uma vez, minha gratidão.

Cordialmente,
PJMS



Em sex., 8 de abr. de 2022 às 13:02, Claudio Buffara <
claudio.buff...@gmail.com> escreveu:

> A volta é fácil também: ao calcular a representação decimal de a/b (a e b
> naturais), nas divisões sucessivas por b só existem b-1 restos possíveis
> (resto = 0 em alguma etapa implica numa decimal finita) e, portanto, após
> não mais do que b-1 divisões, um resto vai se repetir, marcando o início de
> um novo período na representação decimal.
>
> Agora, suponha que  X =
> 0,123456789112233445566778899111222333444555666777888999... seja racional.
> Então existirão n e p naturais tais que, a partir da n-ésima casa decimal
> (1/10^n), os algarismos de X vão se repetir numa sequência com período p.
>
> Mas, pela lei de formação de X, vai existir uma sequência de n+p+1
> algarismos iguais a 1, e esta sequência vai começar após a n-ésima casa
> decimal.
> Ou seja, a sequência vai estar incluída na parte periódica da
> representação decimal de X.
> Mas como o período é p, isso implica que a parte periódica teria que
> ser 111..11 (p algarismos 1) ==> contradição à lei de formação de X.
>
> []s,
> Claudio.
>
>
> On Fri, Apr 8, 2022 at 11:17 AM Pedro José  wrote:
>
>> Bom dia!
>> Posso concluir que um número representado por uma infinidade de
>> algarismos decimais é racional se e somente se tem um período de repetições
>> desses algarismos?
>> A ida é fácil se tiver o período é racional.
>> Já a volta não sei se é verdade e se for há como provar?
>>
>> Meu objetivo primário é saber se:
>> 0,123456789112233445566778899111222333444555666777888999... é racional.
>> As reticências se referem ao aumento de mais um algarismo repetido a cada
>> sequência, ou seja a primeira aparição de 1 será 1, a 2a 11 a 3a 111 e
>> assim sucessivamente, o mesmo vale para os demais algarismos.
>>
>> Alguém poderia me ajudar?
>> Grato,
>> PJMS
>>
>> --
>> Esta mensagem foi verificada pelo sistema de antivírus e
>> acredita-se estar livre de perigo.
>
>
> --
> Esta mensagem foi verificada pelo sistema de antivírus e
> acredita-se estar livre de perigo.

-- 
Esta mensagem foi verificada pelo sistema de antiv�rus e
 acredita-se estar livre de perigo.



[obm-l] Re: [obm-l] Dúvida e ajuda.

2022-04-08 Por tôpico Claudio Buffara
A volta é fácil também: ao calcular a representação decimal de a/b (a e b
naturais), nas divisões sucessivas por b só existem b-1 restos possíveis
(resto = 0 em alguma etapa implica numa decimal finita) e, portanto, após
não mais do que b-1 divisões, um resto vai se repetir, marcando o início de
um novo período na representação decimal.

Agora, suponha que  X =
0,123456789112233445566778899111222333444555666777888999... seja racional.
Então existirão n e p naturais tais que, a partir da n-ésima casa decimal
(1/10^n), os algarismos de X vão se repetir numa sequência com período p.

Mas, pela lei de formação de X, vai existir uma sequência de n+p+1
algarismos iguais a 1, e esta sequência vai começar após a n-ésima casa
decimal.
Ou seja, a sequência vai estar incluída na parte periódica da representação
decimal de X.
Mas como o período é p, isso implica que a parte periódica teria que
ser 111..11 (p algarismos 1) ==> contradição à lei de formação de X.

[]s,
Claudio.


On Fri, Apr 8, 2022 at 11:17 AM Pedro José  wrote:

> Bom dia!
> Posso concluir que um número representado por uma infinidade de algarismos
> decimais é racional se e somente se tem um período de repetições desses
> algarismos?
> A ida é fácil se tiver o período é racional.
> Já a volta não sei se é verdade e se for há como provar?
>
> Meu objetivo primário é saber se:
> 0,123456789112233445566778899111222333444555666777888999... é racional. As
> reticências se referem ao aumento de mais um algarismo repetido a cada
> sequência, ou seja a primeira aparição de 1 será 1, a 2a 11 a 3a 111 e
> assim sucessivamente, o mesmo vale para os demais algarismos.
>
> Alguém poderia me ajudar?
> Grato,
> PJMS
>
> --
> Esta mensagem foi verificada pelo sistema de antivírus e
> acredita-se estar livre de perigo.

-- 
Esta mensagem foi verificada pelo sistema de antiv�rus e
 acredita-se estar livre de perigo.



[obm-l] Re: [obm-l] Dúvida e ajuda.

2022-04-08 Por tôpico Caio Costa
Para a volta considere a repetição dividida por 9...9 onde há o mesmo
número de algarismos na repetição e no denominador, incluindo possíveis
zeros à esquerda.

Exemplo

0.3520012001200120012...

= 0.352 + (0012/)/1000

Em sex., 8 de abr. de 2022 11:17, Pedro José  escreveu:

> Bom dia!
> Posso concluir que um número representado por uma infinidade de algarismos
> decimais é racional se e somente se tem um período de repetições desses
> algarismos?
> A ida é fácil se tiver o período é racional.
> Já a volta não sei se é verdade e se for há como provar?
>
> Meu objetivo primário é saber se:
> 0,123456789112233445566778899111222333444555666777888999... é racional. As
> reticências se referem ao aumento de mais um algarismo repetido a cada
> sequência, ou seja a primeira aparição de 1 será 1, a 2a 11 a 3a 111 e
> assim sucessivamente, o mesmo vale para os demais algarismos.
>
> Alguém poderia me ajudar?
> Grato,
> PJMS
>
> --
> Esta mensagem foi verificada pelo sistema de antivírus e
> acredita-se estar livre de perigo.

-- 
Esta mensagem foi verificada pelo sistema de antiv�rus e
 acredita-se estar livre de perigo.



[obm-l] Dúvida e ajuda.

2022-04-08 Por tôpico Pedro José
Bom dia!
Posso concluir que um número representado por uma infinidade de algarismos
decimais é racional se e somente se tem um período de repetições desses
algarismos?
A ida é fácil se tiver o período é racional.
Já a volta não sei se é verdade e se for há como provar?

Meu objetivo primário é saber se:
0,123456789112233445566778899111222333444555666777888999... é racional. As
reticências se referem ao aumento de mais um algarismo repetido a cada
sequência, ou seja a primeira aparição de 1 será 1, a 2a 11 a 3a 111 e
assim sucessivamente, o mesmo vale para os demais algarismos.

Alguém poderia me ajudar?
Grato,
PJMS

-- 
Esta mensagem foi verificada pelo sistema de antiv�rus e
 acredita-se estar livre de perigo.



[obm-l] Re: [obm-l] Re: [obm-l] Dúvida

2021-11-22 Por tôpico Pedro José
Boa tarde!

Grato, pela ajuda!
Não conheço.
Vou abrir um leque de estudo para tentar entender!
Valeu a curiosidade, com o que cheguei consegui matar o problema.
Genericamente, consegui que a solução levaria a uma expressão que era um
quadrado perfeito,esse era o objetivo. Só que me deu curiosidade, quanto a
resolução. Vou me enveredar no tema.

Cordialmente,
PJMS.

Em ter., 16 de nov. de 2021 às 17:29, Prof. Douglas Oliveira <
profdouglaso.del...@gmail.com> escreveu:

> Equação de Pell
>
> Em seg., 15 de nov. de 2021 13:36, Pedro José 
> escreveu:
>
>> Boa tarde!
>>
>> Alguém saberia como resolver a seguinte equação:
>>
>> x^2-7y^2=1, x,y em Z?
>>
>> Fiz a-7b=1 e achei a= 8 +7k e b=1 +K
>> Logo fica fácil que para k=-1 funciona x^2=1 e y^2=0.
>> Também funciona para k=8 x^2=64 e y^2=9.
>> Mas não sei nem como achar mais soluções nem como provar que só são essas.
>> Alguém poderia me dar uma orientação?
>>
>> Cordialmente,
>> PJMS
>>
>> --
>> Esta mensagem foi verificada pelo sistema de antivírus e
>> acredita-se estar livre de perigo.
>
>
> --
> Esta mensagem foi verificada pelo sistema de antivírus e
> acredita-se estar livre de perigo.

-- 
Esta mensagem foi verificada pelo sistema de antiv�rus e
 acredita-se estar livre de perigo.



[obm-l] Re: [obm-l] Dúvida

2021-11-16 Por tôpico Prof. Douglas Oliveira
Equação de Pell

Em seg., 15 de nov. de 2021 13:36, Pedro José 
escreveu:

> Boa tarde!
>
> Alguém saberia como resolver a seguinte equação:
>
> x^2-7y^2=1, x,y em Z?
>
> Fiz a-7b=1 e achei a= 8 +7k e b=1 +K
> Logo fica fácil que para k=-1 funciona x^2=1 e y^2=0.
> Também funciona para k=8 x^2=64 e y^2=9.
> Mas não sei nem como achar mais soluções nem como provar que só são essas.
> Alguém poderia me dar uma orientação?
>
> Cordialmente,
> PJMS
>
> --
> Esta mensagem foi verificada pelo sistema de antivírus e
> acredita-se estar livre de perigo.

-- 
Esta mensagem foi verificada pelo sistema de antiv�rus e
 acredita-se estar livre de perigo.



[obm-l] Re: [obm-l] Dúvida

2021-11-16 Por tôpico Ian Barquette
Boa tarde!

Esse tipo de equação que você mandou se chama "Equações de Pell". É uma
equação diofantina, mas da forma x^2 - dy^2 = 1, em que d é um número
positivo e não-quadrado-perfeito. Também busca soluções inteiras para "x" e
"y".

Um matemático provou que esse tipo de equação tem infinitas soluções quando
d segue as restrições.

Alguns motivos dos passos da resolução tem origens em matemática mais
avançada do que sei, por isso vou os omitir.

Resolução:
A equação pode ser reescrita como (x+y*sqrt(d))*(x-y*sqrt(d)) = 1, e
podemos achar as n soluções em função do que é chamado de solução
fundamental ("1ª" solução):

x_n + y_n * sqrt(d) = (x_1 + y_1 * sqrt(d))^n
  ("x_n" significa "x índice n")

Para encontrar a solução fundamental, podemos utilizar das frações
contínuas para sqrt(d), encurtando a fração no final do período, e, assim,
o numerador e o denominador do resultado da fração vão ser o "x" e "y",
respectivamente (utilizando d = 7):

A fração contínua para sqrt(7) é [2;*1,1,1,4*], com repetição na parte em
negrito. (Vou fazer a representação linear, pois é mais complicado entender
na forma de fração)
Encurtando-a para o final do período, fica [2;1,1,1], o que equivale a 8/3.

Portanto, a solução fundamental é (8, 3).

A partir disso, pode-se encontrar os outros infinitos resultados pela
fórmula já mencionada acima:
(x_1 + y_1 * sqrt(d))^n
(8 + 3 * sqrt(7))^n

Note que para alcançar a solução (x, y) é necessário "desconsiderar" a raiz
quadrada de "d" no final, e já que se quer x e y inteiros, a solução
será (±x, ±y)

Exemplos:
(8+3 * sqrt(7))^2 = 64 + 2(24*sqrt(7)) + 63 = 127 + 48*sqrt(7)
Solução2: (±127, ±48)

(8+3 * sqrt(7))^3 = (8 + 3*sqrt(7)) * (127 + 48*sqrt(7)) = 8*127 +
8*48*sqrt(7) + 3*sqrt(7)*127 + 3*sqrt(7)*48*sqrt(7) = 1016 + 384*sqrt(7) +
381*sqrt(7) + 1008 = 2024 + 765*sqrt(7)
Solução3: (±2024, ±765)

Desculpe pelas partes sem muita explicação, mas espero que tenha entendido
como se resolve


Em seg., 15 de nov. de 2021 às 13:36, Pedro José 
escreveu:

> Boa tarde!
>
> Alguém saberia como resolver a seguinte equação:
>
> x^2-7y^2=1, x,y em Z?
>
> Fiz a-7b=1 e achei a= 8 +7k e b=1 +K
> Logo fica fácil que para k=-1 funciona x^2=1 e y^2=0.
> Também funciona para k=8 x^2=64 e y^2=9.
> Mas não sei nem como achar mais soluções nem como provar que só são essas.
> Alguém poderia me dar uma orientação?
>
> Cordialmente,
> PJMS
>
> --
> Esta mensagem foi verificada pelo sistema de antivírus e
> acredita-se estar livre de perigo.

-- 
Esta mensagem foi verificada pelo sistema de antiv�rus e
 acredita-se estar livre de perigo.



[obm-l] Dúvida

2021-11-15 Por tôpico Pedro José
Boa tarde!

Alguém saberia como resolver a seguinte equação:

x^2-7y^2=1, x,y em Z?

Fiz a-7b=1 e achei a= 8 +7k e b=1 +K
Logo fica fácil que para k=-1 funciona x^2=1 e y^2=0.
Também funciona para k=8 x^2=64 e y^2=9.
Mas não sei nem como achar mais soluções nem como provar que só são essas.
Alguém poderia me dar uma orientação?

Cordialmente,
PJMS

-- 
Esta mensagem foi verificada pelo sistema de antiv�rus e
 acredita-se estar livre de perigo.



[obm-l] Re: [obm-l] Re: [obm-l] Re: [obm-l] Dúvida

2019-12-05 Por tôpico Bernardo Freitas Paulo da Costa
Oi Pedro e Pedro, e demais colegas da OBM-L

Eu também nunca lera a definição de elipses através da razão entre as
distâncias.  Achei interessante, porque talvez permita "interpolar"
entre elipses, parábolas e hipérboles.  Mas até hoje, todas as
definições que eu vira de elipses (inclusive a da soma das distâncias)
incluíram círculos.  Mas, como talvez tenha desejado indicar o Pedro
Fonini, o importante é *para que serve a definição*.  No caso das
elipses, é muitas vezes importante incluir os círculos com elas, por
exemplo para o teorema de álgebra linear que ele citou.  Talvez o caso
da definição por razão das distâncias indique um outro caminho, mas aí
minha impressão é que o caso seria que a reta diretriz está no
infinito para os círculos (o que, mais uma vez, reforça a unidade das
cônicas no plano projetivo).

Acho que o mais comum, *hoje em dia*, é definir elipses de forma a
incluir os círculos.  Talvez isto invalide a definição que você deu
via razões, que passa a ser um teorema apenas para as elipses com dois
focos distintos.  Mas, por outro lado, permite generalizar de forma
mais natural outros teoremas para os quais a inclusão dos círculos
como elipses simplifique o enunciado.  Talvez você prefira a definição
por razões, mas voltando às origens das cônicas, onde a "classe" é
determinada pela posição relativa do plano secante com relação ao cone
gerador, acredito que a inclusão dos círculos junto com as elipses
seja totalmente razoável.

Cônicas suaves me parecem um assunto avançado; esta terminologia mesmo
já faz pensar em funções (infinitamente) diferenciáveis, etc, típicas
o ensino superior.  Do ponto de vista da geometria algébrica, uma
cõnica é definida como zeros de um polinômio P(x,y) de grau dois,
então esta "definição" não pode servir para separar quem seja suave e
quem não seja.  Se há uma diferença entre um círculo e duas retas que
se intersectam, não é pela regularidade da função que os define
implicitamente: num caso é x^2 + y^2 = 1, no outro, xy = 0.  O que
acaba servindo é a definição de "variedade suave".  E daí eu estaria
puxando mais ainda para temas universitários...

On Wed, Dec 4, 2019 at 9:33 PM Pedro José  wrote:
>
> Boa noite!
> As retas são cônicas degeneradas. Mas são cônicas.
> Definição de cônica :   Dada duas retas g,l concorrentes (cuja interseção é 
> {V} no |R3 que não sejam perpendiculares e um plano Pi. A interseção desse 
> plano com o cone K, reto de vértice V e eixo l , obtido pela rotação da reta 
> g ao redor do ponto V é uma cônica. Podemos ter uma reta, duas retas ou um 
> ponto como cônicas degeneradas.
> Você poderia até ter mencionado o conjunto vazio que não é uma cônica. 
> x^2+y^2=-1.
> Mas na verdade, eu não me expressei com rigor, o que queria dizer é que se 
> escrevermos a função quadrática F(x,y)= 0, que represente a cônica 
> (degenerada ou não) F(x,y) é suave? Ou as cônicas suaves devem ser não 
> degeneradas apenas?
> Outrossim, discordo do seu argumento "...geralmente é mais útil que as 
> definições dos objetos importantes não excluam os casos particulares.."
> Geralmente não é o balizador e sim a definição.
> 1 não é primo. Pois define-se que um primo deve ter dois divisores positivos 
> e 1 só possui um. Poderia argumentar, na sua linha, os dois divisores 
> coincidentes (os que afirmam é divisível por si e pela unidade)
> O quadrado por definição está claro que é retângulo.
> A definição da elipse é de que a soma das distâncias a dois pontos fixos (e 
> não um) é constante. Aí tem a forçação de se considerar dois como um só. Não 
> existe dois pontos coincidentes. Se são dois são distintos. Podemos 
> representar algo de várias maneiras mas se são iguais é só um, representado 
> de várias maneiras. Qual o cardinal do conjunto de focos de uma elipse, no 
> caso de você aceitar a elipse com um único foco?
> Como é a prova que só existe um vazio. Por hipótese há mais de um vazio, 
> vazio1 e vazio2 e no fim chega-se a conclusão que vazio1 = vazio2 e portanto 
> absurdo.Ora, podemos ter vazios coincidentes.
> Amigo, você afirma: "Nunca vi ninguém definir elipse de uma forma que exclua 
> os círculos."
> Você nem se deu ao trabalho de ler a minha nota, antes de comentar, ou então 
> me corrija se o círculo atende à:
> Lugar geométrico do plano em que a razão entre a distância de um ponto ao 
> foco direito e a distância entre esse ponto e uma reta (diretriz direita) é 
> constante e menor que 1 e igual a excentricidade da cônica.
> Como a excentricidade da circunferência é zero, teríamos que ter um ponto 
> fixo em que a distância de cada ponto da circunferência até esse ponto fosse 
> zero. E se na definição tem foco direito está implícito que há um esquerdo. 
> Vale a definição para foco esquerdo. Só atenderia se considerarmos o ponto 
> como uma circunferência de raio zero. E só para esse caso e ainda aceitarmos 
> que quando há só um foco ele tanto é direito quanto esquerdo. Grato pelos 
> comentários. Mas as dúvida persistem.
>
> 

[obm-l] Re: [obm-l] Re: [obm-l] Dúvida

2019-12-04 Por tôpico Pedro José
Boa noite!
As retas são cônicas degeneradas. Mas são cônicas.
Definição de cônica :   Dada duas retas g,l concorrentes (cuja interseção é
{V} no |R3 que não sejam perpendiculares e um plano Pi. A interseção desse
plano com o cone K, reto de vértice V e eixo l , obtido pela rotação da
reta g ao redor do ponto V é uma cônica. Podemos ter uma reta, duas retas
ou um ponto como cônicas degeneradas.
Você poderia até ter mencionado o conjunto vazio que não é uma cônica.
x^2+y^2=-1.
Mas na verdade, eu não me expressei com rigor, o que queria dizer é que se
escrevermos a função quadrática F(x,y)= 0, que represente a cônica
(degenerada ou não) F(x,y) é suave? Ou as cônicas suaves devem ser não
degeneradas apenas?
Outrossim, discordo do seu argumento "...geralmente é mais útil que as
definições dos objetos importantes não excluam os casos particulares.."
Geralmente não é o balizador e sim a definição.
1 não é primo. Pois define-se que um primo deve ter dois divisores
positivos e 1 só possui um. Poderia argumentar, na sua linha, os dois
divisores coincidentes (os que afirmam é divisível por si e pela unidade)
O quadrado por definição está claro que é retângulo.
A definição da elipse é de que a soma das distâncias a dois pontos fixos (e
não um) é constante. Aí tem a forçação de se considerar dois como um só.
Não existe dois pontos coincidentes. Se são dois são distintos. Podemos
representar algo de várias maneiras mas se são iguais é só um, representado
de várias maneiras. Qual o cardinal do conjunto de focos de uma elipse, no
caso de você aceitar a elipse com um único foco?
Como é a prova que só existe um vazio. Por hipótese há mais de um vazio,
vazio1 e vazio2 e no fim chega-se a conclusão que vazio1 = vazio2 e
portanto absurdo.Ora, podemos ter vazios coincidentes.
Amigo, você afirma: "*Nunca vi ninguém definir elipse de uma forma que
exclua os círculos*."
Você nem se deu ao trabalho de ler a minha nota, antes de comentar, ou
então me corrija se o círculo atende à:
Lugar geométrico do plano em que a razão entre a distância de um ponto ao
foco direito e a distância entre esse ponto e uma reta (diretriz direita) é
constante e menor que 1 e igual a excentricidade da cônica.
Como a excentricidade da circunferência é zero, teríamos que ter um ponto
fixo em que a distância de cada ponto da circunferência até esse ponto
fosse zero. E se na definição tem foco direito está implícito que há um
esquerdo. Vale a definição para foco esquerdo. Só atenderia se
considerarmos o ponto como uma circunferência de raio zero. E só para esse
caso e ainda aceitarmos que quando há só um foco ele tanto é direito quanto
esquerdo. Grato pelos comentários. Mas as dúvida persistem.

Saudações,
PJMS



Em qua., 4 de dez. de 2019 às 19:59, Pedro Angelo 
escreveu:

> Em matemática, geralmente é mais útil que as definições dos objetos
> importantes não excluam os casos particulares. Um quadrado é um
> retângulo? Se vc quiser que a definição de "retângulo" inclua somente
> quadriláteros com ângulos retos que não sejam quadrados, vc tem que
> explicitar a parte do "não sejam quadrados" na definição. A definição
> mais simples, "retângulo é um quadrilátero cujos ângulos são todos
> retos" (como o nome já diz!) inclui o quadrado como caso especial. Uma
> coisa parecida ocorre com a elipse. Se vc quiser excluir o círculo, vc
> teria que especificar na definição que vc quer focos distintos. A
> definição mais simples, que cita os focos como sendo "dois pontos", ao
> invés de "dois pontos distintos", inclui o círculo como caso especial.
> E é útill que inclua mesmo. Por exemplo, se vc pensar o círculo como
> sendo um tipo especial de elipse, vc pode enunciar o seguinte teorema:
> "A imagem de uma elipse por uma transformação afim é outra elipse."
> Mas se vc achar que um círculo não é uma elipse, então o teorema (da
> forma que foi enunciado) não vale mais. A questão é que praticamente
> qualquer propriedade interessante apresentada por "elipses
> não-circulares" também será compartilhada pelos círculos. É raro em
> matemática vc precisar de uma elipse que seja proibida de ser um
> círculo. Nunca vi ninguém definir elipse de uma forma que exclua os
> círculos.
>
> Sobre a suavidade: da forma que vc escreveu, eu diria que está um
> pouco ruim. Por exemplo, a função
> F(x,y)=x^2-y^2
> é uma função suave (vc consegue calcular dF/dx e dF/dy, por exemplo).
> Mas vc diria que a equação F(x,y)=0 é uma "cônica suave"? Repare que
> essa equação descreve duas retas que se cruzam na origem. Outras
> funções problemáticas são F(x,y)=x^2+y^2 e F(x,y)=0.
>
> Se F(x,y) é um polinômio de segundo grau em x e y, então F(x,y)=0 é
> uma cônica, e eu diria que essa cônica é "suave" se nenhum dos pontos
> dela (pontos (x,y) tais que F(x,y)=0) satisfaz ao mesmo tempo dF/dx=0
> e dF/dy=0. O fato de pelo menos uma das derivadas parciais de F ser
> não-nula garante que não encontraremos problemas como os do parágrafo
> acima.
>
> abraços!
>
>
> Le mer. 4 déc. 2019 à 19:10, 

[obm-l] Re: [obm-l] Dúvida

2019-12-04 Por tôpico Pedro Angelo
Em matemática, geralmente é mais útil que as definições dos objetos
importantes não excluam os casos particulares. Um quadrado é um
retângulo? Se vc quiser que a definição de "retângulo" inclua somente
quadriláteros com ângulos retos que não sejam quadrados, vc tem que
explicitar a parte do "não sejam quadrados" na definição. A definição
mais simples, "retângulo é um quadrilátero cujos ângulos são todos
retos" (como o nome já diz!) inclui o quadrado como caso especial. Uma
coisa parecida ocorre com a elipse. Se vc quiser excluir o círculo, vc
teria que especificar na definição que vc quer focos distintos. A
definição mais simples, que cita os focos como sendo "dois pontos", ao
invés de "dois pontos distintos", inclui o círculo como caso especial.
E é útill que inclua mesmo. Por exemplo, se vc pensar o círculo como
sendo um tipo especial de elipse, vc pode enunciar o seguinte teorema:
"A imagem de uma elipse por uma transformação afim é outra elipse."
Mas se vc achar que um círculo não é uma elipse, então o teorema (da
forma que foi enunciado) não vale mais. A questão é que praticamente
qualquer propriedade interessante apresentada por "elipses
não-circulares" também será compartilhada pelos círculos. É raro em
matemática vc precisar de uma elipse que seja proibida de ser um
círculo. Nunca vi ninguém definir elipse de uma forma que exclua os
círculos.

Sobre a suavidade: da forma que vc escreveu, eu diria que está um
pouco ruim. Por exemplo, a função
F(x,y)=x^2-y^2
é uma função suave (vc consegue calcular dF/dx e dF/dy, por exemplo).
Mas vc diria que a equação F(x,y)=0 é uma "cônica suave"? Repare que
essa equação descreve duas retas que se cruzam na origem. Outras
funções problemáticas são F(x,y)=x^2+y^2 e F(x,y)=0.

Se F(x,y) é um polinômio de segundo grau em x e y, então F(x,y)=0 é
uma cônica, e eu diria que essa cônica é "suave" se nenhum dos pontos
dela (pontos (x,y) tais que F(x,y)=0) satisfaz ao mesmo tempo dF/dx=0
e dF/dy=0. O fato de pelo menos uma das derivadas parciais de F ser
não-nula garante que não encontraremos problemas como os do parágrafo
acima.

abraços!


Le mer. 4 déc. 2019 à 19:10, Pedro José  a écrit :
>
> Boa noite!
> Estou dando uma repassada nas cônicas para auxiliar um filho de um amigo.
> Dúvidas quanto à cônicas.
> Alguns trabalhos até de mestrandos apontam a circunferência como sendo uma 
> elipse, um caso particular.
> Aprendera que o limite de uma elipse quando a distância entre os focos 
> tendesse para zero era uma circunferência, não obstante a circunferência não 
> é uma elipse.
> A elipse tem dois focos. O que não ocorre na circunferência.
> A elipse pode ser definida como o lugar geométrico do plano em que a razão 
> entre a distância de um ponto ao foco direito e a distância entre esse ponto 
> e uma reta (diretriz direita) é constante e menor que 1 e igual a 
> excentricidade da cônica.
> A circunferência não suporta tal definição.
> Vejo muitos autores chamarem cônicas suaves.Significa que se escrevermos uma 
> equação quadrática com F(x,y)=0 a função F(x,y) é suave?
>
> Grato!
>
> Saudações,
> PJMS.
>
> --
> Esta mensagem foi verificada pelo sistema de antivírus e
> acredita-se estar livre de perigo.

-- 
Esta mensagem foi verificada pelo sistema de antiv�rus e
 acredita-se estar livre de perigo.


=
Instru��es para entrar na lista, sair da lista e usar a lista em
http://www.mat.puc-rio.br/~obmlistas/obm-l.html
=


[obm-l] Dúvida

2019-12-04 Por tôpico Pedro José
Boa noite!
Estou dando uma repassada nas cônicas para auxiliar um filho de um amigo.
Dúvidas quanto à cônicas.
Alguns trabalhos até de mestrandos apontam a circunferência como sendo uma
elipse, um caso particular.
Aprendera que o limite de uma elipse quando a distância entre os focos
tendesse para zero era uma circunferência, não obstante a circunferência
não é uma elipse.
A elipse tem dois focos. O que não ocorre na circunferência.
A elipse pode ser definida como o lugar geométrico do plano em que a razão
entre a distância de um ponto ao foco direito e a distância entre esse
ponto e uma reta (diretriz direita) é constante e menor que 1 e igual a
excentricidade da cônica.
A circunferência não suporta tal definição.
Vejo muitos autores chamarem cônicas suaves.Significa que se escrevermos
uma equação quadrática com F(x,y)=0 a função F(x,y) é suave?

Grato!

Saudações,
PJMS.

-- 
Esta mensagem foi verificada pelo sistema de antiv�rus e
 acredita-se estar livre de perigo.



[obm-l] Re: [obm-l] Re: [obm-l] Re: [obm-l] Dúvida basica equação polar

2019-09-02 Por tôpico Ralph Teixeira
Pois bem, se voce parametrizar com relacao ao centro, teria
x(teta)=1+cos(teta) e y(teta)=sin(teta). Se fosse assim, teria que ser
0 wrote:

> Caro Ralf, obrigado pela resposta.Para mim ficou confuso pq pensei que a
> parametrização do círculo se daria colocando como referencia o novo centro
> do mesmo. Quando penso em circulos diferentes , por exemplo residindo em
> apenas um quadrante tenho dificuldade de imaginar varrendo todos os pontos
> . Vou refletir sobre esses casos pois parecem ser obtidos como vc disse de
> fato.
>
> Att.Gabriel
>
> Em Seg, 2 de set de 2019 18:04, Ralph Teixeira 
> escreveu:
>
>> Bom, vale a pena fazer uma figura primeiro... Fez? Note como este circulo
>> estah nos primeiro e quarto quadrantes apenas.
>>
>> Entao suponho que voce fez as contas e descobriu que r=2cos(teta). No
>> quarto quadrante vale -pi/2> onde pi/2> terceiro quadrante). Por isso que -pi/2>
>> Para ser mas exato, o que aconteceria na equacao r=2cos(teta) para teta
>> entre pi/2 e 3pi/2... Tipo, experimente pensar em teta=pi para fazer um
>> exemplo. Jogando na equacao, ficaria r=2cos(pi)=-2?!?
>>
>> Aqui ha duas opcoes:
>> a) Alguns livros vao insistir que r>=0 sempre. Neste caso, fica claro que
>> pi/2> cosseno fica negativo.
>> b) Alguns outros livros sao mais "liberais" e permitem r<0 -- a
>> interpretacao seria que quando r eh negativo voce anda na reta que forma
>> angulo teta com o eixo x NO SENTIDO OPOSTO. Por exemplo, r=-2 e teta=pi eh
>> de fato o ponto (2,0) (voce anda na direcao NEGATIVA do eixo x, mas voce
>> anda -2, entao acaba andando para a DIREITA duas unidades). Neste caso,
>> colocar teta=pi dah um ponto no circulo sim senhor! Mas, mesmo assim, eu
>> usaria apenas -pi/2> ponto (2,0) JAH APARECEU com teta=0, e nao vejo porque conta-lo duas vezes
>> (e, dependendo da aplicacao, voce NAO QUER contar cada ponto duas vezes).
>>
>> Abraco, Ralph.
>>
>> On Mon, Sep 2, 2019 at 4:55 PM Gabriel Lopes  wrote:
>>
>>> Boa tarde, tenho uma duvida básica da representação em equação polar do
>>> círculo  (x-1)^2 +y^2= 1.
>>>
>>> Pq os intervalo de teta é de -pi/2 a pi/2 e nao de 0 a 2pi?
>>>
>>> --
>>> Esta mensagem foi verificada pelo sistema de antivírus e
>>> acredita-se estar livre de perigo.
>>
>>
>> --
>> Esta mensagem foi verificada pelo sistema de antivírus e
>> acredita-se estar livre de perigo.
>
>
> --
> Esta mensagem foi verificada pelo sistema de antivírus e
> acredita-se estar livre de perigo.

-- 
Esta mensagem foi verificada pelo sistema de antiv�rus e
 acredita-se estar livre de perigo.



[obm-l] Re: [obm-l] Re: [obm-l] Dúvida basica equação polar

2019-09-02 Por tôpico Gabriel Lopes
Caro Ralf, obrigado pela resposta.Para mim ficou confuso pq pensei que a
parametrização do círculo se daria colocando como referencia o novo centro
do mesmo. Quando penso em circulos diferentes , por exemplo residindo em
apenas um quadrante tenho dificuldade de imaginar varrendo todos os pontos
. Vou refletir sobre esses casos pois parecem ser obtidos como vc disse de
fato.

Att.Gabriel

Em Seg, 2 de set de 2019 18:04, Ralph Teixeira  escreveu:

> Bom, vale a pena fazer uma figura primeiro... Fez? Note como este circulo
> estah nos primeiro e quarto quadrantes apenas.
>
> Entao suponho que voce fez as contas e descobriu que r=2cos(teta). No
> quarto quadrante vale -pi/2 onde pi/2 terceiro quadrante). Por isso que -pi/2
> Para ser mas exato, o que aconteceria na equacao r=2cos(teta) para teta
> entre pi/2 e 3pi/2... Tipo, experimente pensar em teta=pi para fazer um
> exemplo. Jogando na equacao, ficaria r=2cos(pi)=-2?!?
>
> Aqui ha duas opcoes:
> a) Alguns livros vao insistir que r>=0 sempre. Neste caso, fica claro que
> pi/2 cosseno fica negativo.
> b) Alguns outros livros sao mais "liberais" e permitem r<0 -- a
> interpretacao seria que quando r eh negativo voce anda na reta que forma
> angulo teta com o eixo x NO SENTIDO OPOSTO. Por exemplo, r=-2 e teta=pi eh
> de fato o ponto (2,0) (voce anda na direcao NEGATIVA do eixo x, mas voce
> anda -2, entao acaba andando para a DIREITA duas unidades). Neste caso,
> colocar teta=pi dah um ponto no circulo sim senhor! Mas, mesmo assim, eu
> usaria apenas -pi/2 ponto (2,0) JAH APARECEU com teta=0, e nao vejo porque conta-lo duas vezes
> (e, dependendo da aplicacao, voce NAO QUER contar cada ponto duas vezes).
>
> Abraco, Ralph.
>
> On Mon, Sep 2, 2019 at 4:55 PM Gabriel Lopes  wrote:
>
>> Boa tarde, tenho uma duvida básica da representação em equação polar do
>> círculo  (x-1)^2 +y^2= 1.
>>
>> Pq os intervalo de teta é de -pi/2 a pi/2 e nao de 0 a 2pi?
>>
>> --
>> Esta mensagem foi verificada pelo sistema de antivírus e
>> acredita-se estar livre de perigo.
>
>
> --
> Esta mensagem foi verificada pelo sistema de antivírus e
> acredita-se estar livre de perigo.

-- 
Esta mensagem foi verificada pelo sistema de antiv�rus e
 acredita-se estar livre de perigo.



[obm-l] Re: [obm-l] Dúvida basica equação polar

2019-09-02 Por tôpico Alexandre Antunes
Boa tarde,

Esse intervalo é arbitrário e pode ser definido para cada problema.
Nessa questão está descrevendo uma curva nesse intervalo.


Em Seg, 2 de set de 2019 16:55, Gabriel Lopes 
escreveu:

> Boa tarde, tenho uma duvida básica da representação em equação polar do
> círculo  (x-1)^2 +y^2= 1.
>
> Pq os intervalo de teta é de -pi/2 a pi/2 e nao de 0 a 2pi?
>
> --
> Esta mensagem foi verificada pelo sistema de antivírus e
> acredita-se estar livre de perigo.

-- 
Esta mensagem foi verificada pelo sistema de antiv�rus e
 acredita-se estar livre de perigo.



[obm-l] Re: [obm-l] Dúvida basica equação polar

2019-09-02 Por tôpico Ralph Teixeira
Bom, vale a pena fazer uma figura primeiro... Fez? Note como este circulo
estah nos primeiro e quarto quadrantes apenas.

Entao suponho que voce fez as contas e descobriu que r=2cos(teta). No
quarto quadrante vale -pi/2=0 sempre. Neste caso, fica claro que
pi/2 wrote:

> Boa tarde, tenho uma duvida básica da representação em equação polar do
> círculo  (x-1)^2 +y^2= 1.
>
> Pq os intervalo de teta é de -pi/2 a pi/2 e nao de 0 a 2pi?
>
> --
> Esta mensagem foi verificada pelo sistema de antivírus e
> acredita-se estar livre de perigo.

-- 
Esta mensagem foi verificada pelo sistema de antiv�rus e
 acredita-se estar livre de perigo.



[obm-l] Dúvida basica equação polar

2019-09-02 Por tôpico Gabriel Lopes
Boa tarde, tenho uma duvida básica da representação em equação polar do
círculo  (x-1)^2 +y^2= 1.

Pq os intervalo de teta é de -pi/2 a pi/2 e nao de 0 a 2pi?

-- 
Esta mensagem foi verificada pelo sistema de antiv�rus e
 acredita-se estar livre de perigo.



[obm-l] Re: [obm-l] Re: [obm-l] Re: [obm-l] Dúvida

2019-05-24 Por tôpico Anderson Torres
Em dom, 19 de mai de 2019 às 13:24, Pedro José 
escreveu:

> Bom dia!
> Anderson,
> obrigado. Porém faltou-me saber se os entendimentos anteriores estão
> corretos.
>

O texto não tinha nenhum glossário para ajudar, ou uma referência do
gênero? Alguns bons livros de Teoria dos Números, em especial os grossões
como o do Hua Loo-Keng, costumam dedicar a primeira folha a convenções.

Eu até suspeito que esses teus entendimentos estejam corretos - dado que
são bem parecidos com outros já consagrados pelo uso - mas não bato o
martelo porque não tenho o contexto.



> Grato,
> PJMS
>
> Em sáb, 18 de mai de 2019 13:27, Anderson Torres <
> torres.anderson...@gmail.com escreveu:
>
>>
>>
>> Em sex, 17 de mai de 2019 às 10:49, Pedro José 
>> escreveu:
>>
>>> Bom dia!
>>>
>>> Tenho uma dúvida sobre os simbolismos, que aparecem recorrentemente, em
>>> artigos sobre teoria dos números, mas que não encontro a definição :
>>> Z[i]/(α) - Entendi como o conjunto das classes de equivalências mod α
>>> em Z{i}
>>> Z[i]/(α)* - Entendi como as classes de equivalência mod α em Z[i], que
>>> são inversíveis.
>>>
>>> Tentando compreender uma demonstração de que todos os números que não
>>> podem ser escritos da forma 4^k(8m+7) com k,m>=0, aceitam ser escritos como
>>> a soma de três parcelas, todas quadrados, me deparei com  [image:
>>> image.png] . O que significa?
>>>
>>
>> Algo como números da forma a+b*sqrt(m) onde a e b são racionais.
>>
>>
>>
>>>
>>> Os outros entendimentos estão corretos?
>>>
>>> Saudações,
>>> PJMS
>>>
>>>
>>> --
>>> Esta mensagem foi verificada pelo sistema de antivírus e
>>> acredita-se estar livre de perigo.
>>
>>
>> --
>> Esta mensagem foi verificada pelo sistema de antivírus e
>> acredita-se estar livre de perigo.
>
>
> --
> Esta mensagem foi verificada pelo sistema de antivírus e
> acredita-se estar livre de perigo.

-- 
Esta mensagem foi verificada pelo sistema de antiv�rus e
 acredita-se estar livre de perigo.



[obm-l] Re: [obm-l] Re: [obm-l] Dúvida

2019-05-19 Por tôpico Pedro José
Bom dia!
Anderson,
obrigado. Porém faltou-me saber se os entendimentos anteriores estão
corretos.

Grato,
PJMS

Em sáb, 18 de mai de 2019 13:27, Anderson Torres <
torres.anderson...@gmail.com escreveu:

>
>
> Em sex, 17 de mai de 2019 às 10:49, Pedro José 
> escreveu:
>
>> Bom dia!
>>
>> Tenho uma dúvida sobre os simbolismos, que aparecem recorrentemente, em
>> artigos sobre teoria dos números, mas que não encontro a definição :
>> Z[i]/(α) - Entendi como o conjunto das classes de equivalências mod α em
>> Z{i}
>> Z[i]/(α)* - Entendi como as classes de equivalência mod α em Z[i], que
>> são inversíveis.
>>
>> Tentando compreender uma demonstração de que todos os números que não
>> podem ser escritos da forma 4^k(8m+7) com k,m>=0, aceitam ser escritos como
>> a soma de três parcelas, todas quadrados, me deparei com  [image:
>> image.png] . O que significa?
>>
>
> Algo como números da forma a+b*sqrt(m) onde a e b são racionais.
>
>
>
>>
>> Os outros entendimentos estão corretos?
>>
>> Saudações,
>> PJMS
>>
>>
>> --
>> Esta mensagem foi verificada pelo sistema de antivírus e
>> acredita-se estar livre de perigo.
>
>
> --
> Esta mensagem foi verificada pelo sistema de antivírus e
> acredita-se estar livre de perigo.

-- 
Esta mensagem foi verificada pelo sistema de antiv�rus e
 acredita-se estar livre de perigo.



[obm-l] Re: [obm-l] Dúvida

2019-05-18 Por tôpico Anderson Torres
Em sex, 17 de mai de 2019 às 10:49, Pedro José 
escreveu:

> Bom dia!
>
> Tenho uma dúvida sobre os simbolismos, que aparecem recorrentemente, em
> artigos sobre teoria dos números, mas que não encontro a definição :
> Z[i]/(α) - Entendi como o conjunto das classes de equivalências mod α em
> Z{i}
> Z[i]/(α)* - Entendi como as classes de equivalência mod α em Z[i], que
> são inversíveis.
>
> Tentando compreender uma demonstração de que todos os números que não
> podem ser escritos da forma 4^k(8m+7) com k,m>=0, aceitam ser escritos como
> a soma de três parcelas, todas quadrados, me deparei com  [image:
> image.png] . O que significa?
>

Algo como números da forma a+b*sqrt(m) onde a e b são racionais.



>
> Os outros entendimentos estão corretos?
>
> Saudações,
> PJMS
>
>
> --
> Esta mensagem foi verificada pelo sistema de antivírus e
> acredita-se estar livre de perigo.

-- 
Esta mensagem foi verificada pelo sistema de antiv�rus e
 acredita-se estar livre de perigo.



[obm-l] Dúvida

2019-05-17 Por tôpico Pedro José
Bom dia!

Tenho uma dúvida sobre os simbolismos, que aparecem recorrentemente, em
artigos sobre teoria dos números, mas que não encontro a definição :
Z[i]/(α) - Entendi como o conjunto das classes de equivalências mod α em
Z{i}
Z[i]/(α)* - Entendi como as classes de equivalência mod α em Z[i], que são
inversíveis.

Tentando compreender uma demonstração de que todos os números que não podem
ser escritos da forma 4^k(8m+7) com k,m>=0, aceitam ser escritos como a
soma de três parcelas, todas quadrados, me deparei com  [image: image.png]
. O que significa?

Os outros entendimentos estão corretos?

Saudações,
PJMS

-- 
Esta mensagem foi verificada pelo sistema de antiv�rus e
 acredita-se estar livre de perigo.



[obm-l] Re: [obm-l] Re: [obm-l] dúvida sobre a OBMU

2019-01-20 Por tôpico Pedro Soares
Combinatória aproveita bastante.
E pra exemplificar o que pode ter em comum, esse ano o problema 6 do Nível
U também estava na prova do nível 3 (não sei o número do problema)

On Sat, 19 Jan 2019 at 09:42, Anderson Torres 
wrote:

> Em sáb, 12 de jan de 2019 às 16:41, Luiz Kv
>  escreveu:
> >
> > Olá, boa tarde, tudo bom ?
> >
> > Gostaria de saber quais conteúdos caem na OBMU diferentes do nível 3 da
> OBM
>
> Acho que, bem, tudo! Dificilmente tem algo que se aproveite
> diretamente. No máximo Combinatória, já vi uma questão de Combinatória
> que poderia ser facilmente dada para o Nível 3.
>
> > Quais as melhores fontes para estudar ?
>
> As provas antigas, as provas da IMC e provas de Universitárias ao
> redor do mundo, como a Putnam dos EUA.
>
> E algumas revistas internacionais :)
>
> > Existe idade máxima para a OBMU ?
>
> Não exatamente, Acho que só cobram ser estudante universitário.
> Algumas competições internacionais cobram idade, porém.
>
>
> >
> > Obrigado, até mais :)
> >
> > --
> > Esta mensagem foi verificada pelo sistema de antivírus e
> > acredita-se estar livre de perigo.
>
> --
> Esta mensagem foi verificada pelo sistema de antivírus e
>  acredita-se estar livre de perigo.
>
>
> =
> Instru�ões para entrar na lista, sair da lista e usar a lista em
> http://www.mat.puc-rio.br/~obmlistas/obm-l.html
> =
>

-- 
Esta mensagem foi verificada pelo sistema de antiv�rus e
 acredita-se estar livre de perigo.



[obm-l] Re: [obm-l] dúvida sobre a OBMU

2019-01-19 Por tôpico Anderson Torres
Em sáb, 12 de jan de 2019 às 16:41, Luiz Kv
 escreveu:
>
> Olá, boa tarde, tudo bom ?
>
> Gostaria de saber quais conteúdos caem na OBMU diferentes do nível 3 da OBM

Acho que, bem, tudo! Dificilmente tem algo que se aproveite
diretamente. No máximo Combinatória, já vi uma questão de Combinatória
que poderia ser facilmente dada para o Nível 3.

> Quais as melhores fontes para estudar ?

As provas antigas, as provas da IMC e provas de Universitárias ao
redor do mundo, como a Putnam dos EUA.

E algumas revistas internacionais :)

> Existe idade máxima para a OBMU ?

Não exatamente, Acho que só cobram ser estudante universitário.
Algumas competições internacionais cobram idade, porém.


>
> Obrigado, até mais :)
>
> --
> Esta mensagem foi verificada pelo sistema de antivírus e
> acredita-se estar livre de perigo.

-- 
Esta mensagem foi verificada pelo sistema de antiv�rus e
 acredita-se estar livre de perigo.


=
Instru��es para entrar na lista, sair da lista e usar a lista em
http://www.mat.puc-rio.br/~obmlistas/obm-l.html
=


[obm-l] dúvida sobre a OBMU

2019-01-12 Por tôpico Luiz Kv
Olá, boa tarde, tudo bom ?

Gostaria de saber quais conteúdos caem na OBMU diferentes do nível 3 da OBM
Quais as melhores fontes para estudar ?
Existe idade máxima para a OBMU ?

Obrigado, até mais :)

-- 
Esta mensagem foi verificada pelo sistema de antiv�rus e
 acredita-se estar livre de perigo.



[obm-l] Re: [obm-l] Dúvida conceitual (equações)

2018-10-16 Por tôpico Eduardo Wagner
Considere multiplicidades.

Em dom, 14 de out de 2018 às 06:38, Vanderlei Nemitz 
escreveu:

> Bom dia!
> Na seguinte questão, que me foi apresentada por um aluno, a resposta
> proposta é a alternativa C (1/2). Eu sempre pensei que apenas
> considerávamos multiplicidades em equações polinomiais. Como essa é uma
> equação exponencial, obtive a resposta B (-1/2). O que é correto pensar?
>
> O produto das raízes da equação 16.4^3x - 40.4^2x + 17.4^x - 2 = 0 é igual
> a:
> A) 1
> B) - 0,5
> C) 0,5
> D) - 1
> E) 0
>
> Muito obrigado!
>
> --
> Esta mensagem foi verificada pelo sistema de antivírus e
> acredita-se estar livre de perigo.

-- 
Esta mensagem foi verificada pelo sistema de antiv�rus e
 acredita-se estar livre de perigo.



[obm-l] Re: [obm-l] Re: [obm-l] Dúvida conceitual (equações)

2018-10-15 Por tôpico Pedro José
Boa tarde!
Artur, não sou contrário a multiplicidade da raiz. Porém, mesmo coma a
multiplicidade, a raiz continua sendo única.
Todavia,não há como negar, facilita sobremaneira as relações de Girard,
para soma e produto é fácil de ajeitar, mas quando passamos a somatório de
produtos dois a dois, três a três... ficaria complicado.

Saudações,
PJMS

Em seg, 15 de out de 2018 às 12:45, Artur Steiner <
artur.costa.stei...@gmail.com> escreveu:

> Isso de se considerar multiplicidades no número de raízes de um polinômio
> é uma convenção conveniente. Facilita muito no caso, por exemplo, das
> famosas relações de Girard. Elas só funcionam se considerarmos as
> multiplicidades. Em análise complexa há também vários teoremas relativos a
> funções analíticas que contam os zeros da função contando multiplicidades.
>
> É claro, por exemplo, que o conjunto de zeros (ou raízes) da função f(x) =
> x^3  é {0}. É uma única raiz com multiplicidade 3. Mas em muitas aplicações
> é mais conveniente supor que são 3 raízes iguais a 0.Sem esquecer que esta
> f só se anula para x = 0.
>
> Há muitas convenções convenientes na matemática. Por exemplo, embora a
> soma seja uma operação binária, convenciona-se que uma soma de uma única
> parcela é a própria parcela. Isto facilita muito.
>
> Artur Costa Steiner
>
> Em dom, 14 de out de 2018 06:33, Vanderlei Nemitz 
> escreveu:
>
>> Bom dia!
>> Na seguinte questão, que me foi apresentada por um aluno, a resposta
>> proposta é a alternativa C (1/2). Eu sempre pensei que apenas
>> considerávamos multiplicidades em equações polinomiais. Como essa é uma
>> equação exponencial, obtive a resposta B (-1/2). O que é correto pensar?
>>
>> O produto das raízes da equação 16.4^3x - 40.4^2x + 17.4^x - 2 = 0 é
>> igual a:
>> A) 1
>> B) - 0,5
>> C) 0,5
>> D) - 1
>> E) 0
>>
>> Muito obrigado!
>>
>> --
>> Esta mensagem foi verificada pelo sistema de antivírus e
>> acredita-se estar livre de perigo.
>
>
> --
> Esta mensagem foi verificada pelo sistema de antivírus e
> acredita-se estar livre de perigo.

-- 
Esta mensagem foi verificada pelo sistema de antiv�rus e
 acredita-se estar livre de perigo.



[obm-l] Re: [obm-l] Dúvida conceitual (equações)

2018-10-15 Por tôpico Artur Steiner
Isso de se considerar multiplicidades no número de raízes de um polinômio é
uma convenção conveniente. Facilita muito no caso, por exemplo, das famosas
relações de Girard. Elas só funcionam se considerarmos as multiplicidades.
Em análise complexa há também vários teoremas relativos a funções
analíticas que contam os zeros da função contando multiplicidades.

É claro, por exemplo, que o conjunto de zeros (ou raízes) da função f(x) =
x^3  é {0}. É uma única raiz com multiplicidade 3. Mas em muitas aplicações
é mais conveniente supor que são 3 raízes iguais a 0.Sem esquecer que esta
f só se anula para x = 0.

Há muitas convenções convenientes na matemática. Por exemplo, embora a soma
seja uma operação binária, convenciona-se que uma soma de uma única parcela
é a própria parcela. Isto facilita muito.

Artur Costa Steiner

Em dom, 14 de out de 2018 06:33, Vanderlei Nemitz 
escreveu:

> Bom dia!
> Na seguinte questão, que me foi apresentada por um aluno, a resposta
> proposta é a alternativa C (1/2). Eu sempre pensei que apenas
> considerávamos multiplicidades em equações polinomiais. Como essa é uma
> equação exponencial, obtive a resposta B (-1/2). O que é correto pensar?
>
> O produto das raízes da equação 16.4^3x - 40.4^2x + 17.4^x - 2 = 0 é igual
> a:
> A) 1
> B) - 0,5
> C) 0,5
> D) - 1
> E) 0
>
> Muito obrigado!
>
> --
> Esta mensagem foi verificada pelo sistema de antivírus e
> acredita-se estar livre de perigo.

-- 
Esta mensagem foi verificada pelo sistema de antiv�rus e
 acredita-se estar livre de perigo.



[obm-l] Re: [obm-l] Re: [obm-l] Re: [obm-l] Re: [obm-l] Dúvida conceitual (equações)

2018-10-15 Por tôpico Bernardo Freitas Paulo da Costa
On Mon, Oct 15, 2018 at 8:07 AM Claudio Buffara
 wrote:
>
> Derivando e igualando a zero o lado esquerdo da sua equação, ficamos com:
> -2*cos(x)*sen(x) + sen(x) = 0 ==>
> sen(x) = 0  ou  cos(x) = 1/2 ==>
> x = 0 ou x = pi ou x = 2pi
> ou x = pi/3 ou x = 5pi/3.
>
> Assim, uma definição que me parece adequado para equações em geral (e não 
> necessariamente polinomiais) da forma f(x) = 0 é que uma raiz de 
> multiplicidade n é raiz de f, f’, ... , f^(n-1) mas não é raiz de f^(n).
>
> Naturalmente, se f não tiver todas as derivadas, precisaremos achar uma 
> definição diferente. Mas talvez, neste caso, nem faça sentido falar em 
> multiplicidade de uma raiz.

Essa definição funciona relativamente bem se f é analítica, porque o
comportamento local é determinado por inteiros.  Se f for apenas
diferenciável, talvez seja complicado dizer algo, como o exemplo
clássico de exp(-1/x^2).  A raiz tem multiplicidade infinita?

Enfim, existem, como você falou, boas razões para incorporar
multiplicidade (por exemplo estabilidade numérica), mas isso em geral
só faz sentido no mundo analítico, onde a noção de "grau" é dada pelas
derivadas.  Acho que mesmo no mundo C-infinito já pode haver
problemas, mas não sou especialista (nessas :D) patologias.  A questão
original, incluindo multiplicidades, pode ser resolvida simplesmente
usando as relações de Girard, que dependem de forma simples da
equação.

Vou tentar dar um exemplo que ilustra meu ponto de vista:  qual o
produto das raízes da equação x^2 - 4x + c?  "Qualquer um" dirá "c".
Mas, naturalmente, se c = 4, a única solução é x=2, e portanto (sem
usar multiplicidades) este produto seria apenas 2.  E daí a fórmula
fica muito mais complicada, com um caso especial, e descontínua.  A
grande sacada do Girard foi, justamente, propor incorporar as
multiplicidades, para simplificar as fórmulas (além, é claro, de
incluir também as soluções negativas, antes consideradas como
"absurdas" - este foi, provavelmente, o maior motivo de as pessoas
considerarem raízes negativas como algo que fazia sentido, e portanto
os números negativos também).  Mas isso não quer dizer que a equação
x^2 - 4x + 4 tenha duas soluções.  É apenas uma forma mais conveniente
de interpretar as raízes quando se pensam nas relações de Girard (e
várias outras fórmulas).  Neste sentido, acho que este tipo de questão
mais atrapalha (porque "era só para usar a fórmula") - a menos que,
justamente, se discuta *porque* falamos de multiplicidade: para que as
fórmulas fiquem mais simples (e você pode incluir "bonitas" também,
por minha conta).  Nada mais.  E esta "simplificação" do entendimento
através da simplificação das fórmulas não se justifica sempre: este
mesmo debate sobre multiplicidades leva a considerar objetos no
infinito (para que todas as retas se intersectem sempre em um ponto),
complexos (para x^2 + 1 = 0 ter raiz), etc.  Muitas vezes, é útil ter
esse entendimento unificado, onde tudo "só depende do grau".  Mas será
mesmo que se eu perguntar para você "em quantos pontos a reta x=3
corta a parábola y=x^2?" você vai dizer "2, é óbvio"?

Abraços,
-- 
Bernardo Freitas Paulo da Costa

-- 
Esta mensagem foi verificada pelo sistema de antiv�rus e
 acredita-se estar livre de perigo.


=
Instru��es para entrar na lista, sair da lista e usar a lista em
http://www.mat.puc-rio.br/~obmlistas/obm-l.html
=


Re: [obm-l] Re: [obm-l] Re: [obm-l] Dúvida conceitual (equações)

2018-10-15 Por tôpico Kevin Felipe Kühl Oliveira
Exatamente nisso que estava pensando. Se fizessemos 4^x = y teriamos uma 
equação polinomial de grau 3, ai fica mais evidente a existência de múltiplas 
raizes.

Abraços

Kevin Kühl
On 15 Oct 2018 07:25 -0300, Claudio Buffara , wrote:
> Qual a soma das raizes de (2^x - 8)^3 = 0?
> Se a equação acima fosse apresentada como:
> 2^(3x) - 24*2^(2x) + 192*2^x - 512 = 0,
> isso mudaria sua resposta?
>
> Enviado do meu iPhone
>
> Em 15 de out de 2018, à(s) 00:29, Vanderlei Nemitz  
> escreveu:
>
> > Valeu, Pedro! Tomara que mais alguém emita sua opinião.
> > Um abraço!
> >
> > > Em dom, 14 de out de 2018 18:59, Pedro José  
> > > escreveu:
> > > > Boa noite!
> > > > Bom questionamento. Vou me posicionar na arquibancada.Â
> > > > Minha posição é controversa. Se quer se levar em conta a repetição 
> > > > tem que se falar do produto das raízes, cada elevada a sua 
> > > > multiplicidade. No caso de soma, cada raiz multiplicada pela 
> > > > multiplicidade.
> > > > Para esse exemplo, o conjunto solução é {1/2,-1} então o produto é 
> > > > -1/2.
> > > > Em suma, não aceito n raízes iguais, mas sim uma raiz de 
> > > > multiplicidade n.
> > > > Se quando queremos provar que algo é unico supomos a existência de 
> > > > dois e provamos que são iguais. Creio que seja contraditório dois ou 
> > > > nais iguais.
> > > > Mas vamos observar as diversas posições, pois, creio que o assunto 
> > > > não seja pacífico.Â
> > > > Saudações,Â
> > > > PJMSÂ
> > > >
> > > > > Em Dom, 14 de out de 2018 06:33, Vanderlei Nemitz 
> > > > >  escreveu:
> > > > > > Bom dia!
> > > > > > Na seguinte questão, que me foi apresentada por um aluno, a 
> > > > > > resposta proposta é a alternativa C (1/2). Eu sempre pensei que 
> > > > > > apenas considerávamos multiplicidades em equações polinomiais. 
> > > > > > Como essa é uma equação exponencial, obtive a resposta B (-1/2). 
> > > > > > O que é correto pensar?
> > > > > >
> > > > > > O produto das raízes da equação 16.4^3x - 40.4^2x + 17.4^x - 2 = 
> > > > > > 0 é igual a:
> > > > > > A) 1
> > > > > > B) - 0,5
> > > > > > C) 0,5
> > > > > > D) - 1
> > > > > > E) 0
> > > > > >
> > > > > > Muito obrigado!
> > > > > >
> > > > > > --
> > > > > > Esta mensagem foi verificada pelo sistema de antivírus e
> > > > > > acredita-se estar livre de perigo.
> > > >
> > > > --
> > > > Esta mensagem foi verificada pelo sistema de antivírus e
> > > > acredita-se estar livre de perigo.
> >
> > --
> > Esta mensagem foi verificada pelo sistema de antivírus e
> > acredita-se estar livre de perigo.
>
> --
> Esta mensagem foi verificada pelo sistema de antiv�rus e
> acredita-se estar livre de perigo.

-- 
Esta mensagem foi verificada pelo sistema de antiv�rus e
 acredita-se estar livre de perigo.



Re: [obm-l] Re: [obm-l] Re: [obm-l] Re: [obm-l] Dúvida conceitual (equações)

2018-10-15 Por tôpico Claudio Buffara
Derivando e igualando a zero o lado esquerdo da sua equação, ficamos com:
-2*cos(x)*sen(x) + sen(x) = 0 ==>
sen(x) = 0  ou  cos(x) = 1/2 ==>
x = 0 ou x = pi ou x = 2pi
ou x = pi/3 ou x = 5pi/3.

Assim, uma definição que me parece adequado para equações em geral (e não 
necessariamente polinomiais) da forma f(x) = 0 é que uma raiz de multiplicidade 
n é raiz de f, f’, ... , f^(n-1) mas não é raiz de f^(n).

Naturalmente, se f não tiver todas as derivadas, precisaremos achar uma 
definição diferente. Mas talvez, neste caso, nem faça sentido falar em 
multiplicidade de uma raiz.

Enviado do meu iPhone

Em 15 de out de 2018, à(s) 08:13, Vanderlei Nemitz  
escreveu:

> Claudio:
> Eu ficaria com a mesma dúvida!
> Pensaria em apenas uma raiz.
> 
> Qual é a soma das raízes da equação (cos x)^2 - cos x + 1/4 = 0 no 
> intervalo [0, 2pi]?
> 
> Em seg, 15 de out de 2018 07:00, Claudio Buffara  
> escreveu:
>> Qual a soma das raizes de (2^x - 8)^3 = 0?
>> Se a equação acima fosse apresentada como:
>> 2^(3x) - 24*2^(2x) + 192*2^x - 512 = 0,
>> isso mudaria sua resposta?
>> 
>> Enviado do meu iPhone
>> 
>> Em 15 de out de 2018, Ã (s) 00:29, Vanderlei Nemitz  
>> escreveu:
>> 
>>> Valeu, Pedro! Tomara que mais alguém emita sua opinião.
>>> Um abraço!
>>> 
>>> Em dom, 14 de out de 2018 18:59, Pedro José  
>>> escreveu:
 Boa noite!
 Bom questionamento. Vou me posicionar na arquibancada. 
 Minha posição é controversa. Se quer se levar em conta a 
 repetição tem que se falar do produto das raízes, cada elevada a 
 sua multiplicidade. No caso de soma, cada raiz multiplicada pela 
 multiplicidade.
 Para esse exemplo, o conjunto solução é {1/2,-1} então o 
 produto é -1/2.
 Em suma, não aceito n raízes iguais, mas sim uma raiz de 
 multiplicidade n.
 Se quando queremos provar que algo é unico supomos a existência de 
 dois e provamos que são iguais. Creio que seja contraditório dois ou 
 nais iguais.
 Mas vamos observar as diversas posições, pois, creio que o assunto 
 não seja pacífico. 
 Saudações, 
 PJMS 
 
 Em Dom, 14 de out de 2018 06:33, Vanderlei Nemitz  
 escreveu:
> Bom dia!
> Na seguinte questão, que me foi apresentada por um aluno, a resposta 
> proposta é a alternativa C (1/2). Eu sempre pensei que apenas 
> considerávamos multiplicidades em equações polinomiais. Como 
> essa é uma equação exponencial, obtive a resposta B (-1/2). O 
> que é correto pensar?
> 
> O produto das raízes da equação 16.4^3x - 40.4^2x + 17.4^x - 2 = 
> 0 é igual a:
> A) 1
> B) - 0,5
> C) 0,5
> D) - 1
> E) 0
> 
> Muito obrigado!
> 
> -- 
> Esta mensagem foi verificada pelo sistema de antivírus e 
> acredita-se estar livre de perigo.
 
 -- 
 Esta mensagem foi verificada pelo sistema de antivírus e 
 acredita-se estar livre de perigo.
>>> 
>>> -- 
>>> Esta mensagem foi verificada pelo sistema de antivírus e 
>>> acredita-se estar livre de perigo.
>> 
>> -- 
>> Esta mensagem foi verificada pelo sistema de antivírus e 
>> acredita-se estar livre de perigo.
> 
> -- 
> Esta mensagem foi verificada pelo sistema de antivírus e 
> acredita-se estar livre de perigo.

-- 
Esta mensagem foi verificada pelo sistema de antiv�rus e
 acredita-se estar livre de perigo.



Re: [obm-l] Re: [obm-l] Re: [obm-l] Re: [obm-l] Dúvida conceitual (equações)

2018-10-15 Por tôpico Claudio Buffara
Pensando só como uma equação, talvez faça sentido não considerar a 
multiplicidade.

Mas, no seu exemplo, no intervalo [0,2pi], os gráficos de 
f(x) = cos(x) - 1/2 
e de
g(x) = (cos(x) - 1/2)^2
tem um comportamento bem distinto um do outro em vizinhanças de pi/3 e 5pi/3.
Por exemplo, o gráfico de f corta o eixo x em pi/3 enquanto que o de g apenas 
tangencia o eixo neste ponto.
Idem pros outros exemplos.
Isso sugere que, mesmo nestes casos, talvez seja conveniente considerar a 
multiplicidade de uma raiz.

Enviado do meu iPhone

Em 15 de out de 2018, à(s) 08:13, Vanderlei Nemitz  
escreveu:

> Claudio:
> Eu ficaria com a mesma dúvida!
> Pensaria em apenas uma raiz.
> 
> Qual é a soma das raízes da equação (cos x)^2 - cos x + 1/4 = 0 no 
> intervalo [0, 2pi]?
> 
> Em seg, 15 de out de 2018 07:00, Claudio Buffara  
> escreveu:
>> Qual a soma das raizes de (2^x - 8)^3 = 0?
>> Se a equação acima fosse apresentada como:
>> 2^(3x) - 24*2^(2x) + 192*2^x - 512 = 0,
>> isso mudaria sua resposta?
>> 
>> Enviado do meu iPhone
>> 
>> Em 15 de out de 2018, Ã (s) 00:29, Vanderlei Nemitz  
>> escreveu:
>> 
>>> Valeu, Pedro! Tomara que mais alguém emita sua opinião.
>>> Um abraço!
>>> 
>>> Em dom, 14 de out de 2018 18:59, Pedro José  
>>> escreveu:
 Boa noite!
 Bom questionamento. Vou me posicionar na arquibancada. 
 Minha posição é controversa. Se quer se levar em conta a 
 repetição tem que se falar do produto das raízes, cada elevada a 
 sua multiplicidade. No caso de soma, cada raiz multiplicada pela 
 multiplicidade.
 Para esse exemplo, o conjunto solução é {1/2,-1} então o 
 produto é -1/2.
 Em suma, não aceito n raízes iguais, mas sim uma raiz de 
 multiplicidade n.
 Se quando queremos provar que algo é unico supomos a existência de 
 dois e provamos que são iguais. Creio que seja contraditório dois ou 
 nais iguais.
 Mas vamos observar as diversas posições, pois, creio que o assunto 
 não seja pacífico. 
 Saudações, 
 PJMS 
 
 Em Dom, 14 de out de 2018 06:33, Vanderlei Nemitz  
 escreveu:
> Bom dia!
> Na seguinte questão, que me foi apresentada por um aluno, a resposta 
> proposta é a alternativa C (1/2). Eu sempre pensei que apenas 
> considerávamos multiplicidades em equações polinomiais. Como 
> essa é uma equação exponencial, obtive a resposta B (-1/2). O 
> que é correto pensar?
> 
> O produto das raízes da equação 16.4^3x - 40.4^2x + 17.4^x - 2 = 
> 0 é igual a:
> A) 1
> B) - 0,5
> C) 0,5
> D) - 1
> E) 0
> 
> Muito obrigado!
> 
> -- 
> Esta mensagem foi verificada pelo sistema de antivírus e 
> acredita-se estar livre de perigo.
 
 -- 
 Esta mensagem foi verificada pelo sistema de antivírus e 
 acredita-se estar livre de perigo.
>>> 
>>> -- 
>>> Esta mensagem foi verificada pelo sistema de antivírus e 
>>> acredita-se estar livre de perigo.
>> 
>> -- 
>> Esta mensagem foi verificada pelo sistema de antivírus e 
>> acredita-se estar livre de perigo.
> 
> -- 
> Esta mensagem foi verificada pelo sistema de antivírus e 
> acredita-se estar livre de perigo.

-- 
Esta mensagem foi verificada pelo sistema de antiv�rus e
 acredita-se estar livre de perigo.



[obm-l] Re: [obm-l] Re: [obm-l] Re: [obm-l] Dúvida conceitual (equações)

2018-10-15 Por tôpico Vanderlei Nemitz
Claudio:
Eu ficaria com a mesma dúvida!
Pensaria em apenas uma raiz.

Qual é a soma das raízes da equação (cos x)^2 - cos x + 1/4 = 0 no
intervalo [0, 2pi]?

Em seg, 15 de out de 2018 07:00, Claudio Buffara 
escreveu:

> Qual a soma das raizes de (2^x - 8)^3 = 0?
> Se a equação acima fosse apresentada como:
> 2^(3x) - 24*2^(2x) + 192*2^x - 512 = 0,
> isso mudaria sua resposta?
>
> Enviado do meu iPhone
>
> Em 15 de out de 2018, à(s) 00:29, Vanderlei Nemitz 
> escreveu:
>
> Valeu, Pedro! Tomara que mais alguém emita sua opinião.
> Um abraço!
>
> Em dom, 14 de out de 2018 18:59, Pedro José 
> escreveu:
>
>> Boa noite!
>> Bom questionamento. Vou me posicionar na arquibancada.Â
>> Minha posição é controversa. Se quer se levar em conta a repetição
>> tem que se falar do produto das raízes, cada elevada a sua multiplicidade.
>> No caso de soma, cada raiz multiplicada pela multiplicidade.
>> Para esse exemplo, o conjunto solução é {1/2,-1} então o produto é
>> -1/2.
>> Em suma, não aceito n raízes iguais, mas sim uma raiz de multiplicidade
>> n.
>> Se quando queremos provar que algo é unico supomos a existência de dois
>> e provamos que são iguais. Creio que seja contraditório dois ou nais
>> iguais.
>> Mas vamos observar as diversas posições, pois, creio que o assunto não
>> seja pacífico.Â
>> Saudações,Â
>> PJMSÂ
>>
>> Em Dom, 14 de out de 2018 06:33, Vanderlei Nemitz 
>> escreveu:
>>
>>> Bom dia!
>>> Na seguinte questão, que me foi apresentada por um aluno, a resposta
>>> proposta é a alternativa C (1/2). Eu sempre pensei que apenas
>>> considerávamos multiplicidades em equações polinomiais. Como essa é uma
>>> equação exponencial, obtive a resposta B (-1/2). O que é correto pensar?
>>>
>>> O produto das raízes da equação 16.4^3x - 40.4^2x + 17.4^x - 2 = 0 é
>>> igual a:
>>> A) 1
>>> B) - 0,5
>>> C) 0,5
>>> D) - 1
>>> E) 0
>>>
>>> Muito obrigado!
>>>
>>> --
>>> Esta mensagem foi verificada pelo sistema de antivírus e
>>> acredita-se estar livre de perigo.
>>
>>
>> --
>> Esta mensagem foi verificada pelo sistema de antivírus e
>> acredita-se estar livre de perigo.
>
>
> --
> Esta mensagem foi verificada pelo sistema de antivírus e
> acredita-se estar livre de perigo.
>
>
> --
> Esta mensagem foi verificada pelo sistema de antivírus e
> acredita-se estar livre de perigo.
>

-- 
Esta mensagem foi verificada pelo sistema de antiv�rus e
 acredita-se estar livre de perigo.



Re: [obm-l] Re: [obm-l] Re: [obm-l] Dúvida conceitual (equações)

2018-10-15 Por tôpico Claudio Buffara
Qual a soma das raizes de (2^x - 8)^3 = 0?
Se a equação acima fosse apresentada como:
2^(3x) - 24*2^(2x) + 192*2^x - 512 = 0,
isso mudaria sua resposta?

Enviado do meu iPhone

Em 15 de out de 2018, à(s) 00:29, Vanderlei Nemitz  
escreveu:

> Valeu, Pedro! Tomara que mais alguém emita sua opinião.
> Um abraço!
> 
> Em dom, 14 de out de 2018 18:59, Pedro José  escreveu:
>> Boa noite!
>> Bom questionamento. Vou me posicionar na arquibancada. 
>> Minha posição é controversa. Se quer se levar em conta a repetição tem 
>> que se falar do produto das raízes, cada elevada a sua multiplicidade. No 
>> caso de soma, cada raiz multiplicada pela multiplicidade.
>> Para esse exemplo, o conjunto solução é {1/2,-1} então o produto é -1/2.
>> Em suma, não aceito n raízes iguais, mas sim uma raiz de multiplicidade n.
>> Se quando queremos provar que algo é unico supomos a existência de dois e 
>> provamos que são iguais. Creio que seja contraditório dois ou nais iguais.
>> Mas vamos observar as diversas posições, pois, creio que o assunto não 
>> seja pacífico. 
>> Saudações, 
>> PJMSÂ 
>> 
>> Em Dom, 14 de out de 2018 06:33, Vanderlei Nemitz  
>> escreveu:
>>> Bom dia!
>>> Na seguinte questão, que me foi apresentada por um aluno, a resposta 
>>> proposta é a alternativa C (1/2). Eu sempre pensei que apenas 
>>> considerávamos multiplicidades em equações polinomiais. Como essa é uma 
>>> equação exponencial, obtive a resposta B (-1/2). O que é correto pensar?
>>> 
>>> O produto das raízes da equação 16.4^3x - 40.4^2x + 17.4^x - 2 = 0 é 
>>> igual a:
>>> A) 1
>>> B) - 0,5
>>> C) 0,5
>>> D) - 1
>>> E) 0
>>> 
>>> Muito obrigado!
>>> 
>>> -- 
>>> Esta mensagem foi verificada pelo sistema de antivírus e 
>>> acredita-se estar livre de perigo.
>> 
>> -- 
>> Esta mensagem foi verificada pelo sistema de antivírus e 
>> acredita-se estar livre de perigo.
> 
> -- 
> Esta mensagem foi verificada pelo sistema de antivírus e 
> acredita-se estar livre de perigo.

-- 
Esta mensagem foi verificada pelo sistema de antiv�rus e
 acredita-se estar livre de perigo.



[obm-l] Re: [obm-l] Re: [obm-l] Dúvida conceitual (equações)

2018-10-14 Por tôpico Vanderlei Nemitz
Valeu, Pedro! Tomara que mais alguém emita sua opinião.
Um abraço!

Em dom, 14 de out de 2018 18:59, Pedro José  escreveu:

> Boa noite!
> Bom questionamento. Vou me posicionar na arquibancada.
> Minha posição é controversa. Se quer se levar em conta a repetição tem que
> se falar do produto das raízes, cada elevada a sua multiplicidade. No caso
> de soma, cada raiz multiplicada pela multiplicidade.
> Para esse exemplo, o conjunto solução é {1/2,-1} então o produto é -1/2.
> Em suma, não aceito n raízes iguais, mas sim uma raiz de multiplicidade n.
> Se quando queremos provar que algo é unico supomos a existência de dois e
> provamos que são iguais. Creio que seja contraditório dois ou nais iguais.
> Mas vamos observar as diversas posições, pois, creio que o assunto não
> seja pacífico.
> Saudações,
> PJMS
>
> Em Dom, 14 de out de 2018 06:33, Vanderlei Nemitz 
> escreveu:
>
>> Bom dia!
>> Na seguinte questão, que me foi apresentada por um aluno, a resposta
>> proposta é a alternativa C (1/2). Eu sempre pensei que apenas
>> considerávamos multiplicidades em equações polinomiais. Como essa é uma
>> equação exponencial, obtive a resposta B (-1/2). O que é correto pensar?
>>
>> O produto das raízes da equação 16.4^3x - 40.4^2x + 17.4^x - 2 = 0 é
>> igual a:
>> A) 1
>> B) - 0,5
>> C) 0,5
>> D) - 1
>> E) 0
>>
>> Muito obrigado!
>>
>> --
>> Esta mensagem foi verificada pelo sistema de antivírus e
>> acredita-se estar livre de perigo.
>
>
> --
> Esta mensagem foi verificada pelo sistema de antivírus e
> acredita-se estar livre de perigo.

-- 
Esta mensagem foi verificada pelo sistema de antiv�rus e
 acredita-se estar livre de perigo.



Re: [obm-l] Re: [obm-l] Dúvida conceitual (equações)

2018-10-14 Por tôpico Jefferson Franca
 Concordo com Pedro
Em domingo, 14 de outubro de 2018 19:51:25 BRT, Pedro José 
 escreveu:  
 
 Boa noite!Bom questionamento. Vou me posicionar na arquibancada. Minha posição 
é controversa. Se quer se levar em conta a repetição tem que se falar do 
produto das raízes, cada elevada a sua multiplicidade. No caso de soma, cada 
raiz multiplicada pela multiplicidade.Para esse exemplo, o conjunto solução é 
{1/2,-1} então o produto é -1/2.Em suma, não aceito n raízes iguais, mas sim 
uma raiz de multiplicidade n.Se quando queremos provar que algo é unico supomos 
a existência de dois e provamos que são iguais. Creio que seja contraditório 
dois ou nais iguais.Mas vamos observar as diversas posições, pois, creio que o 
assunto não seja pacífico. Saudações, PJMS 
Em Dom, 14 de out de 2018 06:33, Vanderlei Nemitz  
escreveu:

Bom dia!Na seguinte questão, que me foi apresentada por um aluno, a resposta 
proposta é a alternativa C (1/2). Eu sempre pensei que apenas considerávamos 
multiplicidades em equações polinomiais. Como essa é uma equação exponencial, 
obtive a resposta B (-1/2). O que é correto pensar?
O produto das raízes da equação 16.4^3x - 40.4^2x + 17.4^x - 2 = 0 é igual a:A) 
1B) - 0,5C) 0,5D) - 1E) 0
Muito obrigado!
--
Esta mensagem foi verificada pelo sistema de antivírus e 
 acredita-se estar livre de perigo.

--
Esta mensagem foi verificada pelo sistema de antiv�rus e 
 acredita-se estar livre de perigo.  
-- 
Esta mensagem foi verificada pelo sistema de antiv�rus e
 acredita-se estar livre de perigo.



[obm-l] Re: [obm-l] Dúvida conceitual (equações)

2018-10-14 Por tôpico Pedro José
Boa noite!
Bom questionamento. Vou me posicionar na arquibancada.
Minha posição é controversa. Se quer se levar em conta a repetição tem que
se falar do produto das raízes, cada elevada a sua multiplicidade. No caso
de soma, cada raiz multiplicada pela multiplicidade.
Para esse exemplo, o conjunto solução é {1/2,-1} então o produto é -1/2.
Em suma, não aceito n raízes iguais, mas sim uma raiz de multiplicidade n.
Se quando queremos provar que algo é unico supomos a existência de dois e
provamos que são iguais. Creio que seja contraditório dois ou nais iguais.
Mas vamos observar as diversas posições, pois, creio que o assunto não seja
pacífico.
Saudações,
PJMS

Em Dom, 14 de out de 2018 06:33, Vanderlei Nemitz 
escreveu:

> Bom dia!
> Na seguinte questão, que me foi apresentada por um aluno, a resposta
> proposta é a alternativa C (1/2). Eu sempre pensei que apenas
> considerávamos multiplicidades em equações polinomiais. Como essa é uma
> equação exponencial, obtive a resposta B (-1/2). O que é correto pensar?
>
> O produto das raízes da equação 16.4^3x - 40.4^2x + 17.4^x - 2 = 0 é igual
> a:
> A) 1
> B) - 0,5
> C) 0,5
> D) - 1
> E) 0
>
> Muito obrigado!
>
> --
> Esta mensagem foi verificada pelo sistema de antivírus e
> acredita-se estar livre de perigo.

-- 
Esta mensagem foi verificada pelo sistema de antiv�rus e
 acredita-se estar livre de perigo.



[obm-l] Dúvida conceitual (equações)

2018-10-14 Por tôpico Vanderlei Nemitz
Bom dia!
Na seguinte questão, que me foi apresentada por um aluno, a resposta
proposta é a alternativa C (1/2). Eu sempre pensei que apenas
considerávamos multiplicidades em equações polinomiais. Como essa é uma
equação exponencial, obtive a resposta B (-1/2). O que é correto pensar?

O produto das raízes da equação 16.4^3x - 40.4^2x + 17.4^x - 2 = 0 é igual
a:
A) 1
B) - 0,5
C) 0,5
D) - 1
E) 0

Muito obrigado!

-- 
Esta mensagem foi verificada pelo sistema de antiv�rus e
 acredita-se estar livre de perigo.



[obm-l] Re: [obm-l] Re: [obm-l] Dúvida

2018-08-23 Por tôpico Pedro José
Boa noite!
É fato.
Grato,
PJMS.

Em Qua, 22 de ago de 2018 23:00, Ralph Teixeira 
escreveu:

> Acho que nao... Ah, se eu entendi corretamente, (3,6,9) e (3,5,12) seria
> um contra-exemplo.
>
> Abraco, Ralph.
>
>
> On Wed, Aug 22, 2018 at 8:06 PM Pedro José  wrote:
>
>> Boa noite.
>>
>> Sejam duas sequências em ordem crescente com ai,bi >0 e k elementos ambas.
>> se:
>> (a1+a2+a3+...+ak)/(b1+b2+b3+...+bk)=a1a2a3a3...ak/(b1b2b3...bk) podemos
>> dizer que
>> ai=bi para 0>
>> Grato,
>> PJMS
>>
>> --
>> Esta mensagem foi verificada pelo sistema de antivírus e
>> acredita-se estar livre de perigo.
>
>
> --
> Esta mensagem foi verificada pelo sistema de antivírus e
> acredita-se estar livre de perigo.

-- 
Esta mensagem foi verificada pelo sistema de antiv�rus e
 acredita-se estar livre de perigo.



[obm-l] Re: [obm-l] Dúvida

2018-08-22 Por tôpico Ralph Teixeira
Acho que nao... Ah, se eu entendi corretamente, (3,6,9) e (3,5,12) seria um
contra-exemplo.

Abraco, Ralph.


On Wed, Aug 22, 2018 at 8:06 PM Pedro José  wrote:

> Boa noite.
>
> Sejam duas sequências em ordem crescente com ai,bi >0 e k elementos ambas.
> se:
> (a1+a2+a3+...+ak)/(b1+b2+b3+...+bk)=a1a2a3a3...ak/(b1b2b3...bk) podemos
> dizer que
> ai=bi para 0
> Grato,
> PJMS
>
> --
> Esta mensagem foi verificada pelo sistema de antivírus e
> acredita-se estar livre de perigo.

-- 
Esta mensagem foi verificada pelo sistema de antiv�rus e
 acredita-se estar livre de perigo.



[obm-l] Dúvida

2018-08-22 Por tôpico Pedro José
Boa noite.

Sejam duas sequências em ordem crescente com ai,bi >0 e k elementos ambas.
se:
(a1+a2+a3+...+ak)/(b1+b2+b3+...+bk)=a1a2a3a3...ak/(b1b2b3...bk) podemos
dizer que
ai=bi para 0

[obm-l] Re: [obm-l] Re: [obm-l] Dúvida

2018-05-31 Por tôpico Pedro José
Bom dia!
Corrigindo uma grande bobagem, confirme me alertado.
A ordem de 10 nos 11 é 2 e não 1. Mas como 2|6, não muda nada.

Saudações,
PJMS

Em Sex, 25 de mai de 2018 14:37, Pedro José  escreveu:

> Boa tarde!
> Creio ter conseguido.
> Criei um número com fatores congruentes a 1 mod 6, exceto o 5 e o11.
> Além disso a ordem de 10 mod desses fatores é sempre 6, exceto o 5 e o 11
> que será 1, melhor. Mas o 5 não tem problema.
> Então o objetivo é firmar um número da seguinte forma:
> ...AB...B...C concatenado com o número criado, mencionado
> anteriormente.
> O número criado foi:
> 84.259.175 = 5^2*7^2*11*13^2*37
> Então a soma dos algarismos desse número é 41 e dos quadrados de seus
> algarismos é 265.
> No número que pretendo formar o número de algarismos em bloco será
> múltiplo de 6.
> Então fica o sistema para apenas dois blocos:
> ax+by= (1001-41)/6=160
> a^2*x +b^2*y=(S2 -265)/6.
> Onde x e y é a quantidade de repetições de blocos de 6 algarismos e a e b
> são os algarismos e S2 é a soma dos quadrados de todos dígitos.
> Agora preciso criar S2 que feche com o problema. Tem que ser 1 mod 6, para
> quando subtrair 265, ser divisível por 6. Deve ser um divisor do número
> criado no início. 5^2*7^2*11*13^2*37.
> Seja S2=5005=5*7*11*13
> xa+yb=160
> xa^2+yb^2= (5005-265)/6=790.
> Como 6| 790 - 160, 1 e 3 formam uma boa escolha, mas infortunadamente, o
> número de blocos de 1 dá negativo.
> Então introduzi 9 blocos de 8 para acertar, já que há liberdade.
>
> Aí dão 9 blocos de 8, 25 blocos de 1 e 21 blocos de três, concatenação ao
> final, 84.259.175.
> É o número fica.
> 10^274*8*(10^54-1)/9+10^124*(10^150-1)/9+10^8*3*(10^126)/9+
> 5^2*7^2*11*13^2*17.
> Como
> 10^6 =1 mod p, com p=7 ou p=11 ou p= 13 e 5 |10, S2=5*7*11*13, S2 divide
> cada parcela e portanto o número.
> O número são 54 algarismos 8, seguidos de 150 algarismos 1,seguidos de126
> algarismos 3 seguidos de 84259175.
> Deve ter um jeito mais elegante de resolver.
> Saudações,
> PJMS
>
> Em Qui, 24 de mai de 2018 23:51, Pedro José 
> escreveu:
>
>> Boa noite!
>> Minha primeira tentativa foi tudo 1. Mas aí a soma dos quadrados também é
>> 1001=7*11*13.
>> As ordens de 10 mod desses fatores são 6, 1 e 6. Mas têm 1001 algarismos
>> e aí 6 ł 1001não serve.
>> Tentei outros arranjos com grupos de algarismos iguais, mas sem sucesso.
>> Mas o que não compreendo é porque não há a divulgação da resposta.
>> Saudações,
>> PJMS
>>
>> Em Qui, 24 de mai de 2018 21:09, Anderson Torres <
>> torres.anderson...@gmail.com> escreveu:
>>
>>> Em 23 de maio de 2018 21:41, Pedro José  escreveu:
>>> > Boa noite!
>>> > Há algum motivo para não disponibilizarem o gabarito da olimpiada de
>>> mayo?
>>> > Gostaria de ver a solução de um problema da XXII olimpiada:
>>> > Dizemos que um número inteiro positivo é qua-divi se é divisível
>>> pela
>>> > soma dos quadrados de seus dígitos, e além disso nenhum de seus
>>> dígitos
>>> > é igual a zero.
>>> > a) Encontre um número qua-divi tal que a soma de seus dígitos seja
>>> 24.
>>> > b) Encontre um número qua-divi tal que a soma de seus dígitos seja
>>> 1001.
>>>
>>> Só jogando uma ideia solta, eu tentaria calcular para casos como
>>> 111...11. A soma dos dígitos é N e o número é (10^N-1)/9
>>>
>>> Se isso não servir, talvez 111..12222 também possa ser
>>> útil.
>>>
>>> >
>>> > Grato.
>>> > Saudações,
>>> > PJMS
>>> >
>>> > --
>>> > Esta mensagem foi verificada pelo sistema de antivírus e
>>> > acredita-se estar livre de perigo.
>>>
>>> --
>>> Esta mensagem foi verificada pelo sistema de antivírus e
>>>  acredita-se estar livre de perigo.
>>>
>>>
>>> =
>>> Instru�ões para entrar na lista, sair da lista e usar a lista em
>>> http://www.mat.puc-rio.br/~obmlistas/obm-l.html
>>> =
>>>
>>

-- 
Esta mensagem foi verificada pelo sistema de antiv�rus e
 acredita-se estar livre de perigo.



[obm-l] Re: [obm-l] Re: [obm-l] Dúvida

2018-05-25 Por tôpico Pedro José
Boa tarde!
Creio ter conseguido.
Criei um número com fatores congruentes a 1 mod 6, exceto o 5 e o11.
Além disso a ordem de 10 mod desses fatores é sempre 6, exceto o 5 e o 11
que será 1, melhor. Mas o 5 não tem problema.
Então o objetivo é firmar um número da seguinte forma:
...AB...B...C concatenado com o número criado, mencionado
anteriormente.
O número criado foi:
84.259.175 = 5^2*7^2*11*13^2*37
Então a soma dos algarismos desse número é 41 e dos quadrados de seus
algarismos é 265.
No número que pretendo formar o número de algarismos em bloco será múltiplo
de 6.
Então fica o sistema para apenas dois blocos:
ax+by= (1001-41)/6=160
a^2*x +b^2*y=(S2 -265)/6.
Onde x e y é a quantidade de repetições de blocos de 6 algarismos e a e b
são os algarismos e S2 é a soma dos quadrados de todos dígitos.
Agora preciso criar S2 que feche com o problema. Tem que ser 1 mod 6, para
quando subtrair 265, ser divisível por 6. Deve ser um divisor do número
criado no início. 5^2*7^2*11*13^2*37.
Seja S2=5005=5*7*11*13
xa+yb=160
xa^2+yb^2= (5005-265)/6=790.
Como 6| 790 - 160, 1 e 3 formam uma boa escolha, mas infortunadamente, o
número de blocos de 1 dá negativo.
Então introduzi 9 blocos de 8 para acertar, já que há liberdade.

Aí dão 9 blocos de 8, 25 blocos de 1 e 21 blocos de três, concatenação ao
final, 84.259.175.
É o número fica.
10^274*8*(10^54-1)/9+10^124*(10^150-1)/9+10^8*3*(10^126)/9+
5^2*7^2*11*13^2*17.
Como
10^6 =1 mod p, com p=7 ou p=11 ou p= 13 e 5 |10, S2=5*7*11*13, S2 divide
cada parcela e portanto o número.
O número são 54 algarismos 8, seguidos de 150 algarismos 1,seguidos de126
algarismos 3 seguidos de 84259175.
Deve ter um jeito mais elegante de resolver.
Saudações,
PJMS

Em Qui, 24 de mai de 2018 23:51, Pedro José  escreveu:

> Boa noite!
> Minha primeira tentativa foi tudo 1. Mas aí a soma dos quadrados também é
> 1001=7*11*13.
> As ordens de 10 mod desses fatores são 6, 1 e 6. Mas têm 1001 algarismos e
> aí 6 ł 1001não serve.
> Tentei outros arranjos com grupos de algarismos iguais, mas sem sucesso.
> Mas o que não compreendo é porque não há a divulgação da resposta.
> Saudações,
> PJMS
>
> Em Qui, 24 de mai de 2018 21:09, Anderson Torres <
> torres.anderson...@gmail.com> escreveu:
>
>> Em 23 de maio de 2018 21:41, Pedro José  escreveu:
>> > Boa noite!
>> > Há algum motivo para não disponibilizarem o gabarito da olimpiada de
>> mayo?
>> > Gostaria de ver a solução de um problema da XXII olimpiada:
>> > Dizemos que um número inteiro positivo é qua-divi se é divisível
>> pela
>> > soma dos quadrados de seus dígitos, e além disso nenhum de seus
>> dígitos
>> > é igual a zero.
>> > a) Encontre um número qua-divi tal que a soma de seus dígitos seja 24.
>> > b) Encontre um número qua-divi tal que a soma de seus dígitos seja
>> 1001.
>>
>> Só jogando uma ideia solta, eu tentaria calcular para casos como
>> 111...11. A soma dos dígitos é N e o número é (10^N-1)/9
>>
>> Se isso não servir, talvez 111..12222 também possa ser
>> útil.
>>
>> >
>> > Grato.
>> > Saudações,
>> > PJMS
>> >
>> > --
>> > Esta mensagem foi verificada pelo sistema de antivírus e
>> > acredita-se estar livre de perigo.
>>
>> --
>> Esta mensagem foi verificada pelo sistema de antivírus e
>>  acredita-se estar livre de perigo.
>>
>>
>> =
>> Instru�ões para entrar na lista, sair da lista e usar a lista em
>> http://www.mat.puc-rio.br/~obmlistas/obm-l.html
>> =
>>
>

-- 
Esta mensagem foi verificada pelo sistema de antiv�rus e
 acredita-se estar livre de perigo.



[obm-l] Re: [obm-l] Re: [obm-l] Dúvida

2018-05-24 Por tôpico Pedro José
Boa noite!
Minha primeira tentativa foi tudo 1. Mas aí a soma dos quadrados também é
1001=7*11*13.
As ordens de 10 mod desses fatores são 6, 1 e 6. Mas têm 1001 algarismos e
aí 6 ł 1001não serve.
Tentei outros arranjos com grupos de algarismos iguais, mas sem sucesso.
Mas o que não compreendo é porque não há a divulgação da resposta.
Saudações,
PJMS

Em Qui, 24 de mai de 2018 21:09, Anderson Torres <
torres.anderson...@gmail.com> escreveu:

> Em 23 de maio de 2018 21:41, Pedro José  escreveu:
> > Boa noite!
> > Há algum motivo para não disponibilizarem o gabarito da olimpiada de
> mayo?
> > Gostaria de ver a solução de um problema da XXII olimpiada:
> > Dizemos que um número inteiro positivo é qua-divi se é divisível pela
> > soma dos quadrados de seus dígitos, e além disso nenhum de seus
> dígitos
> > é igual a zero.
> > a) Encontre um número qua-divi tal que a soma de seus dígitos seja 24.
> > b) Encontre um número qua-divi tal que a soma de seus dígitos seja
> 1001.
>
> Só jogando uma ideia solta, eu tentaria calcular para casos como
> 111...11. A soma dos dígitos é N e o número é (10^N-1)/9
>
> Se isso não servir, talvez 111..12222 também possa ser
> útil.
>
> >
> > Grato.
> > Saudações,
> > PJMS
> >
> > --
> > Esta mensagem foi verificada pelo sistema de antivírus e
> > acredita-se estar livre de perigo.
>
> --
> Esta mensagem foi verificada pelo sistema de antivírus e
>  acredita-se estar livre de perigo.
>
>
> =
> Instru�ões para entrar na lista, sair da lista e usar a lista em
> http://www.mat.puc-rio.br/~obmlistas/obm-l.html
> =
>

-- 
Esta mensagem foi verificada pelo sistema de antiv�rus e
 acredita-se estar livre de perigo.



[obm-l] Re: [obm-l] Dúvida

2018-05-24 Por tôpico Anderson Torres
Em 23 de maio de 2018 21:41, Pedro José  escreveu:
> Boa noite!
> Há algum motivo para não disponibilizarem o gabarito da olimpiada de mayo?
> Gostaria de ver a solução de um problema da XXII olimpiada:
> Dizemos que um número inteiro positivo é qua-divi se é divisível pela
> soma dos quadrados de seus dígitos, e além disso nenhum de seus dígitos
> é igual a zero.
> a) Encontre um número qua-divi tal que a soma de seus dígitos seja 24.
> b) Encontre um número qua-divi tal que a soma de seus dígitos seja 1001.

Só jogando uma ideia solta, eu tentaria calcular para casos como
111...11. A soma dos dígitos é N e o número é (10^N-1)/9

Se isso não servir, talvez 111..12222 também possa ser útil.

>
> Grato.
> Saudações,
> PJMS
>
> --
> Esta mensagem foi verificada pelo sistema de antivírus e
> acredita-se estar livre de perigo.

-- 
Esta mensagem foi verificada pelo sistema de antiv�rus e
 acredita-se estar livre de perigo.


=
Instru��es para entrar na lista, sair da lista e usar a lista em
http://www.mat.puc-rio.br/~obmlistas/obm-l.html
=


[obm-l] Dúvida

2018-05-23 Por tôpico Pedro José
Boa noite!
Há algum motivo para não disponibilizarem o gabarito da olimpiada de mayo?
Gostaria de ver a solução de um problema da XXII olimpiada:
Dizemos que um número inteiro positivo é qua-divi se é divisível pela
soma dos quadrados de seus dígitos, e além disso nenhum de seus dígitos
é igual a zero.
a) Encontre um número qua-divi tal que a soma de seus dígitos seja 24.
b) Encontre um número qua-divi tal que a soma de seus dígitos seja 1001.

Grato.
Saudações,
PJMS

-- 
Esta mensagem foi verificada pelo sistema de antiv�rus e
 acredita-se estar livre de perigo.



[obm-l] Re: [obm-l] Re: [obm-l] Re: [obm-l] Re: [obm-l] Dúvida num Enunciado

2018-04-26 Por tôpico Pedro José
Boa tarde!
Bernardo,
Realmente eu falhei. Fiquei com a expressão |x+3| < 4 na cabeça. Até uso um
delta, e comento que não pode ser maior que 4.
Saudações,
PJMS

Em 25 de abr de 2018 22:33, "Jaare Oregim" 
escreveu:

>
>
> 2018-04-25 21:30 GMT-03:00 Bernardo Freitas Paulo da Costa <
> bernardo...@gmail.com>:
>
>> 2018-04-25 20:41 GMT-03:00 Claudio Buffara :
>> > O [...]
>> "Determine r > 0 tal que [ |x+3| < r => (A^2 - 10A + 9 > 0 para todo A
>> real) ]."
>>
>> Que continua com o "problema" de ter um "x" livre.  Daí, a proposição
>> entre colchetes tem um valor (verdadeiro/falso) que depende de x.
>>
>
> se o x tá livre *não* tem valor-verdade. Sentença aberta não tem
> valor-verdade.
>
> tb acho que a intenção é "Determine r > 0 tal que (para todo x real, |x+3|
> < r => x^2 - 10x + 9 > 0)."
>
> --
> Esta mensagem foi verificada pelo sistema de antivírus e
> acredita-se estar livre de perigo.

-- 
Esta mensagem foi verificada pelo sistema de antiv�rus e
 acredita-se estar livre de perigo.



[obm-l] Re: [obm-l] Re: [obm-l] Re: [obm-l] Dúvida num Enunciado

2018-04-25 Por tôpico Jaare Oregim
2018-04-25 21:30 GMT-03:00 Bernardo Freitas Paulo da Costa <
bernardo...@gmail.com>:

> 2018-04-25 20:41 GMT-03:00 Claudio Buffara :
> > O [...]
> "Determine r > 0 tal que [ |x+3| < r => (A^2 - 10A + 9 > 0 para todo A
> real) ]."
>
> Que continua com o "problema" de ter um "x" livre.  Daí, a proposição
> entre colchetes tem um valor (verdadeiro/falso) que depende de x.
>

se o x tá livre *não* tem valor-verdade. Sentença aberta não tem
valor-verdade.

tb acho que a intenção é "Determine r > 0 tal que (para todo x real, |x+3|
< r => x^2 - 10x + 9 > 0)."

-- 
Esta mensagem foi verificada pelo sistema de antiv�rus e
 acredita-se estar livre de perigo.



[obm-l] Re: [obm-l] Re: [obm-l] Re: [obm-l] Re: [obm-l] Dúvida num Enunciado

2018-04-25 Por tôpico Claudio Buffara
Verdade! Reparei agora que deve ser r > 0.
Então provavelmente o "para todo x real" não deveria estar lá.
Neste caso, vira um problema com mais cara de EM:

Achar todos os r > 0 tais que
SE  x pertence ao intervalo (-3-r , -3+r )
ENTÃO  x^2 - 10x + 9 > 0

x^2 - 10x + 9 > 0  sss  x pertence a (-inf,1) união (9,+inf).

Assim, observamos que se 0 < r <= 4, então (-3-r,-3+r) está contido em
(-inf,1).
Logo, se 0 < r <= 4 então a implicação acima é verdadeira.

[]s,
Claudio.




2018-04-25 21:47 GMT-03:00 Pedro José :

> Boa noite!
> Cláudio,
> o problema tem restrição r>0. Não dá para seguir nessa linha de r< 0.
> Saudações,
> PJMS
>
> Em 25 de abr de 2018 21:42, "Bernardo Freitas Paulo da Costa" <
> bernardo...@gmail.com> escreveu:
>
>> 2018-04-25 20:20 GMT-03:00 Pedro José :
>> > Boa tarde!
>> > Realmente o enunciado está mal feito.
>> >
>> > Se |x+3| < r, não pode ser para todo o Real. Na verdade é x pertence a
>> |R.
>> >
>> > x^2 -10x + 9 >0  ==> x pertence a A = (-oo, 1) U (9,oo)
>> >
>> > então temos que escolher r de modo que quando resolvamos |x + 3| < r,
>> tenha
>> > x num subconjunto de A
>> >
>> > x < -3 ==> x+3 < 0 ==> -x -3 < r ==> r > x+3 Se r > 4 vai ter 1=< x =<9
>> > atendendo |x +3| <4 + delta. Portanto x <4
>> > então |x+3| < 4, conferindo
>> > x > -3 ==> x+3 <4  ==> x<1, atende.
>> > se x<-3 atende por hipótese. Mas se quiser conferir. -x - 3 < 4 : -x <
>> 7: x
>> >>7, mas x <-3, não tem solução.
>> >
>> > x>=- 3 ==> x+3>=0 ==> x+3 < r. Se r >=4, existirá solução em [1,9].
>> >
>> > Portanto r pertence a (0,4)
>>
>> Só um detalhe: r = 4 também serve: se |x+3| < 4, temos -7 < x < 1, que
>> está contido em A.
>>
>> A minha forma preferida de resolver este exercício é gráfica:
>> desenhamos o conjunto A, depois tomamos P = -3, e traçamos um
>> intervalo simétrico em P de maior raio possível contido em A.  Dá r <=
>> 4 ou r < 4 (no desenho, é difícil decidir entre o estrito ou não) e
>> daí tem que pensar um pouco para detectar se r = 4 serve.
>>
>> Abraços,
>> --
>> Bernardo Freitas Paulo da Costa
>>
>> --
>> Esta mensagem foi verificada pelo sistema de antivírus e
>>  acredita-se estar livre de perigo.
>>
>>
>> =
>> Instru�ões para entrar na lista, sair da lista e usar a lista em
>> http://www.mat.puc-rio.br/~obmlistas/obm-l.html
>> =
>>
>
> --
> Esta mensagem foi verificada pelo sistema de antivírus e
> acredita-se estar livre de perigo.
>

-- 
Esta mensagem foi verificada pelo sistema de antiv�rus e
 acredita-se estar livre de perigo.



[obm-l] Re: [obm-l] Re: [obm-l] Re: [obm-l] Re: [obm-l] Dúvida num Enunciado

2018-04-25 Por tôpico Luiz Antonio Rodrigues
Olá, Bernardo!
Boa noite!
Vou tentar fazer a resolução graficamente...
Muito obrigado!
Um abraço!
Luiz

On Wed, Apr 25, 2018, 9:55 PM Pedro José  wrote:

> Boa noite!
> Cláudio,
> o problema tem restrição r>0. Não dá para seguir nessa linha de r< 0.
> Saudações,
> PJMS
>
> Em 25 de abr de 2018 21:42, "Bernardo Freitas Paulo da Costa" <
> bernardo...@gmail.com> escreveu:
>
>> 2018-04-25 20:20 GMT-03:00 Pedro José :
>> > Boa tarde!
>> > Realmente o enunciado está mal feito.
>> >
>> > Se |x+3| < r, não pode ser para todo o Real. Na verdade é x pertence a
>> |R.
>> >
>> > x^2 -10x + 9 >0  ==> x pertence a A = (-oo, 1) U (9,oo)
>> >
>> > então temos que escolher r de modo que quando resolvamos |x + 3| < r,
>> tenha
>> > x num subconjunto de A
>> >
>> > x < -3 ==> x+3 < 0 ==> -x -3 < r ==> r > x+3 Se r > 4 vai ter 1=< x =<9
>> > atendendo |x +3| <4 + delta. Portanto x <4
>> > então |x+3| < 4, conferindo
>> > x > -3 ==> x+3 <4  ==> x<1, atende.
>> > se x<-3 atende por hipótese. Mas se quiser conferir. -x - 3 < 4 : -x <
>> 7: x
>> >>7, mas x <-3, não tem solução.
>> >
>> > x>=- 3 ==> x+3>=0 ==> x+3 < r. Se r >=4, existirá solução em [1,9].
>> >
>> > Portanto r pertence a (0,4)
>>
>> Só um detalhe: r = 4 também serve: se |x+3| < 4, temos -7 < x < 1, que
>> está contido em A.
>>
>> A minha forma preferida de resolver este exercício é gráfica:
>> desenhamos o conjunto A, depois tomamos P = -3, e traçamos um
>> intervalo simétrico em P de maior raio possível contido em A.  Dá r <=
>> 4 ou r < 4 (no desenho, é difícil decidir entre o estrito ou não) e
>> daí tem que pensar um pouco para detectar se r = 4 serve.
>>
>> Abraços,
>> --
>> Bernardo Freitas Paulo da Costa
>>
>> --
>> Esta mensagem foi verificada pelo sistema de antivírus e
>>  acredita-se estar livre de perigo.
>>
>>
>> =
>> Instru�ões para entrar na lista, sair da lista e usar a lista em
>> http://www.mat.puc-rio.br/~obmlistas/obm-l.html
>> =
>>
>
> --
> Esta mensagem foi verificada pelo sistema de antivírus e
> acredita-se estar livre de perigo.

-- 
Esta mensagem foi verificada pelo sistema de antiv�rus e
 acredita-se estar livre de perigo.



[obm-l] Re: [obm-l] Re: [obm-l] Re: [obm-l] Dúvida num Enunciado

2018-04-25 Por tôpico Pedro José
Boa noite!
Cláudio,
o problema tem restrição r>0. Não dá para seguir nessa linha de r< 0.
Saudações,
PJMS

Em 25 de abr de 2018 21:42, "Bernardo Freitas Paulo da Costa" <
bernardo...@gmail.com> escreveu:

> 2018-04-25 20:20 GMT-03:00 Pedro José :
> > Boa tarde!
> > Realmente o enunciado está mal feito.
> >
> > Se |x+3| < r, não pode ser para todo o Real. Na verdade é x pertence a
> |R.
> >
> > x^2 -10x + 9 >0  ==> x pertence a A = (-oo, 1) U (9,oo)
> >
> > então temos que escolher r de modo que quando resolvamos |x + 3| < r,
> tenha
> > x num subconjunto de A
> >
> > x < -3 ==> x+3 < 0 ==> -x -3 < r ==> r > x+3 Se r > 4 vai ter 1=< x =<9
> > atendendo |x +3| <4 + delta. Portanto x <4
> > então |x+3| < 4, conferindo
> > x > -3 ==> x+3 <4  ==> x<1, atende.
> > se x<-3 atende por hipótese. Mas se quiser conferir. -x - 3 < 4 : -x <
> 7: x
> >>7, mas x <-3, não tem solução.
> >
> > x>=- 3 ==> x+3>=0 ==> x+3 < r. Se r >=4, existirá solução em [1,9].
> >
> > Portanto r pertence a (0,4)
>
> Só um detalhe: r = 4 também serve: se |x+3| < 4, temos -7 < x < 1, que
> está contido em A.
>
> A minha forma preferida de resolver este exercício é gráfica:
> desenhamos o conjunto A, depois tomamos P = -3, e traçamos um
> intervalo simétrico em P de maior raio possível contido em A.  Dá r <=
> 4 ou r < 4 (no desenho, é difícil decidir entre o estrito ou não) e
> daí tem que pensar um pouco para detectar se r = 4 serve.
>
> Abraços,
> --
> Bernardo Freitas Paulo da Costa
>
> --
> Esta mensagem foi verificada pelo sistema de antivírus e
>  acredita-se estar livre de perigo.
>
>
> =
> Instru�ões para entrar na lista, sair da lista e usar a lista em
> http://www.mat.puc-rio.br/~obmlistas/obm-l.html
> =
>

-- 
Esta mensagem foi verificada pelo sistema de antiv�rus e
 acredita-se estar livre de perigo.



[obm-l] Re: [obm-l] Re: [obm-l] Dúvida num Enunciado

2018-04-25 Por tôpico Bernardo Freitas Paulo da Costa
2018-04-25 20:20 GMT-03:00 Pedro José :
> Boa tarde!
> Realmente o enunciado está mal feito.
>
> Se |x+3| < r, não pode ser para todo o Real. Na verdade é x pertence a |R.
>
> x^2 -10x + 9 >0  ==> x pertence a A = (-oo, 1) U (9,oo)
>
> então temos que escolher r de modo que quando resolvamos |x + 3| < r, tenha
> x num subconjunto de A
>
> x < -3 ==> x+3 < 0 ==> -x -3 < r ==> r > x+3 Se r > 4 vai ter 1=< x =<9
> atendendo |x +3| <4 + delta. Portanto x <4
> então |x+3| < 4, conferindo
> x > -3 ==> x+3 <4  ==> x<1, atende.
> se x<-3 atende por hipótese. Mas se quiser conferir. -x - 3 < 4 : -x < 7: x
>>7, mas x <-3, não tem solução.
>
> x>=- 3 ==> x+3>=0 ==> x+3 < r. Se r >=4, existirá solução em [1,9].
>
> Portanto r pertence a (0,4)

Só um detalhe: r = 4 também serve: se |x+3| < 4, temos -7 < x < 1, que
está contido em A.

A minha forma preferida de resolver este exercício é gráfica:
desenhamos o conjunto A, depois tomamos P = -3, e traçamos um
intervalo simétrico em P de maior raio possível contido em A.  Dá r <=
4 ou r < 4 (no desenho, é difícil decidir entre o estrito ou não) e
daí tem que pensar um pouco para detectar se r = 4 serve.

Abraços,
-- 
Bernardo Freitas Paulo da Costa

-- 
Esta mensagem foi verificada pelo sistema de antiv�rus e
 acredita-se estar livre de perigo.


=
Instru��es para entrar na lista, sair da lista e usar a lista em
http://www.mat.puc-rio.br/~obmlistas/obm-l.html
=


[obm-l] Re: [obm-l] Re: [obm-l] Dúvida num Enunciado

2018-04-25 Por tôpico Bernardo Freitas Paulo da Costa
2018-04-25 20:41 GMT-03:00 Claudio Buffara :
> O consequente (x^2 - 10x + 9 > 0 para todo x real) é falso  (tome qualquer x
> no intervalo [1,9]).
>
> Logo, para a implicação ser verdadeira, o antecedente ( |x+3| < r ) deve ser
> falso, o que ocorre se e somente se r < 0.
>
> É mais ou menos a mesma coisa que (se 1 < 0, então 3+5 = 7), que é uma
> sentença verdadeira (Falso -> Falso é Verdadeiro).

Acho que há duas coisas.  Uma é a interpretação do enunciado.  Alguns
(eu me incluo, e o Pedro José também) vão ler como:

"Determine r > 0 tal que (para todo x real, |x+3| < r => x^2 - 10x + 9 > 0)."

A minha razão principal é porque o x aparece do lado esquerdo da
implicação, e portanto eu fico com a sensação que ele deveria também
estar quantificado.  Mas não é obrigatório.

Ao ler como você fez, a frase fica

"Determine r > 0 tal que [ |x+3| < r => (x^2 - 10x + 9 > 0 para todo x real) ]."

Nesta interpretação (que está mais próxima do texto original...), a
frase entre colchetes tem uma variável livre: o "x", que não está
quantificado.  Para enfatizar, como você mesmo separou o consequente,
ela não mudaria de valor se fosse

"Determine r > 0 tal que [ |x+3| < r => (A^2 - 10A + 9 > 0 para todo A real) ]."

Que continua com o "problema" de ter um "x" livre.  Daí, a proposição
entre colchetes tem um valor (verdadeiro/falso) que depende de x.
Assim, a frase completa "Determine r ..." também depende do valor de
x.  O problema fica bem diferente.  Primeiro, podemos simplificar o
enunciado para

"Determine r > 0 tal que [ |x+3| < r => FALSO ]."

Depois, pegando carona na sua solução, temos que ter o antecedente
falso, para que a afirmação entre colchetes seja verdadeira.  Ou seja:

"Determine r > 0 tal que [ |x+3| < r é FALSO ]."

Re-escrevendo, fica "Determine r > 0 tal que [ |x+3| >= r ].", o que
dá a solução: r <= |x+3|.  Repare que a solução está em função de x,
como esperado, já que o enunciado original também tinha um x livre.
Acho esta interpretação pouco plausível para um exercício, mas acho o
exercício de resolvê-la interessante ;-)

Abraços,
-- 
Bernardo Freitas Paulo da Costa

-- 
Esta mensagem foi verificada pelo sistema de antiv�rus e
 acredita-se estar livre de perigo.


=
Instru��es para entrar na lista, sair da lista e usar a lista em
http://www.mat.puc-rio.br/~obmlistas/obm-l.html
=


[obm-l] Re: [obm-l] Re: [obm-l] Dúvida num Enunciado

2018-04-25 Por tôpico Luiz Antonio Rodrigues
Olá, Claudio!
Boa noite!
Eu não havia percebido que o consequente é falso...
Preciso ficar mais atento!
Muito obrigado pela ajuda!
Um abraço!
Luiz

On Wed, Apr 25, 2018, 8:49 PM Claudio Buffara 
wrote:

> O consequente (x^2 - 10x + 9 > 0 para todo x real) é falso  (tome qualquer
> x no intervalo [1,9]).
>
> Logo, para a implicação ser verdadeira, o antecedente ( |x+3| < r ) deve
> ser falso, o que ocorre se e somente se r < 0.
>
> É mais ou menos a mesma coisa que (se 1 < 0, então 3+5 = 7), que é uma
> sentença verdadeira (Falso -> Falso é Verdadeiro).
>
> []s,
> Claudio.
>
> 2018-04-25 16:41 GMT-03:00 Luiz Antonio Rodrigues :
>
>> Olá, pessoal!
>> Boa tarde!
>> Estou tentando fazer o exercício abaixo, mas o problema é que eu não
>> entendi o enunciado...
>>
>> Determine para quais valores de r (r>0) a implicação é verdadeira:
>> |x+3| x^2 - 10x + 9 > 0 para todo x real.
>>
>> Agradeço qualquer ajuda!
>> Um abraço!
>> Luiz
>>
>> --
>> Esta mensagem foi verificada pelo sistema de antivírus e
>> acredita-se estar livre de perigo.
>
>
>
> --
> Esta mensagem foi verificada pelo sistema de antivírus e
> acredita-se estar livre de perigo.

-- 
Esta mensagem foi verificada pelo sistema de antiv�rus e
 acredita-se estar livre de perigo.



[obm-l] Re: [obm-l] Dúvida num Enunciado

2018-04-25 Por tôpico Claudio Buffara
O consequente (x^2 - 10x + 9 > 0 para todo x real) é falso  (tome qualquer
x no intervalo [1,9]).

Logo, para a implicação ser verdadeira, o antecedente ( |x+3| < r ) deve
ser falso, o que ocorre se e somente se r < 0.

É mais ou menos a mesma coisa que (se 1 < 0, então 3+5 = 7), que é uma
sentença verdadeira (Falso -> Falso é Verdadeiro).

[]s,
Claudio.

2018-04-25 16:41 GMT-03:00 Luiz Antonio Rodrigues :

> Olá, pessoal!
> Boa tarde!
> Estou tentando fazer o exercício abaixo, mas o problema é que eu não
> entendi o enunciado...
>
> Determine para quais valores de r (r>0) a implicação é verdadeira:
> |x+3| x^2 - 10x + 9 > 0 para todo x real.
>
> Agradeço qualquer ajuda!
> Um abraço!
> Luiz
>
> --
> Esta mensagem foi verificada pelo sistema de antivírus e
> acredita-se estar livre de perigo.

-- 
Esta mensagem foi verificada pelo sistema de antiv�rus e
 acredita-se estar livre de perigo.



[obm-l] Re: [obm-l] Re: [obm-l] Dúvida num Enunciado

2018-04-25 Por tôpico Luiz Antonio Rodrigues
Olá, Pedro!
Boa noite!
O resultado é esse mesmo.
Agora eu entendi o que o problema pede.
Muito obrigado!
Um abraço!
Luiz

On Wed, Apr 25, 2018, 8:29 PM Pedro José  wrote:

> Boa tarde!
> Realmente o enunciado está mal feito.
>
> Se |x+3| < r, não pode ser para todo o Real. Na verdade é x pertence a |R.
>
> x^2 -10x + 9 >0  ==> x pertence a A = (-oo, 1) U (9,oo)
>
> então temos que escolher r de modo que quando resolvamos |x + 3| < r,
> tenha x num subconjunto de A
>
> x < -3 ==> x+3 < 0 ==> -x -3 < r ==> r > x+3 Se r > 4 vai ter 1=< x =<9
> atendendo |x +3| <4 + delta. Portanto x <4
> então |x+3| < 4, conferindo
> x > -3 ==> x+3 <4  ==> x<1, atende.
> se x<-3 atende por hipótese. Mas se quiser conferir. -x - 3 < 4 : -x < 7:
> x >7, mas x <-3, não tem solução.
>
> x>=- 3 ==> x+3>=0 ==> x+3 < r. Se r >=4, existirá solução em [1,9].
>
> Portanto r pertence a (0,4)
>
> creio que seja isso.
>
> Saudações,
> PJMS.
>
>
>
> Em 25 de abril de 2018 16:41, Luiz Antonio Rodrigues <
> rodrigue...@gmail.com> escreveu:
>
>> Olá, pessoal!
>> Boa tarde!
>> Estou tentando fazer o exercício abaixo, mas o problema é que eu não
>> entendi o enunciado...
>>
>> Determine para quais valores de r (r>0) a implicação é verdadeira:
>> |x+3| x^2 - 10x + 9 > 0 para todo x real.
>>
>> Agradeço qualquer ajuda!
>> Um abraço!
>> Luiz
>>
>> --
>> Esta mensagem foi verificada pelo sistema de antivírus e
>> acredita-se estar livre de perigo.
>
>
>
> --
> Esta mensagem foi verificada pelo sistema de antivírus e
> acredita-se estar livre de perigo.

-- 
Esta mensagem foi verificada pelo sistema de antiv�rus e
 acredita-se estar livre de perigo.



[obm-l] Re: [obm-l] Dúvida num Enunciado

2018-04-25 Por tôpico Pedro José
Boa tarde!
Realmente o enunciado está mal feito.

Se |x+3| < r, não pode ser para todo o Real. Na verdade é x pertence a |R.

x^2 -10x + 9 >0  ==> x pertence a A = (-oo, 1) U (9,oo)

então temos que escolher r de modo que quando resolvamos |x + 3| < r, tenha
x num subconjunto de A

x < -3 ==> x+3 < 0 ==> -x -3 < r ==> r > x+3 Se r > 4 vai ter 1=< x =<9
atendendo |x +3| <4 + delta. Portanto x <4
então |x+3| < 4, conferindo
x > -3 ==> x+3 <4  ==> x<1, atende.
se x<-3 atende por hipótese. Mas se quiser conferir. -x - 3 < 4 : -x < 7: x
>7, mas x <-3, não tem solução.

x>=- 3 ==> x+3>=0 ==> x+3 < r. Se r >=4, existirá solução em [1,9].

Portanto r pertence a (0,4)

creio que seja isso.

Saudações,
PJMS.



Em 25 de abril de 2018 16:41, Luiz Antonio Rodrigues 
escreveu:

> Olá, pessoal!
> Boa tarde!
> Estou tentando fazer o exercício abaixo, mas o problema é que eu não
> entendi o enunciado...
>
> Determine para quais valores de r (r>0) a implicação é verdadeira:
> |x+3| x^2 - 10x + 9 > 0 para todo x real.
>
> Agradeço qualquer ajuda!
> Um abraço!
> Luiz
>
> --
> Esta mensagem foi verificada pelo sistema de antivírus e
> acredita-se estar livre de perigo.

-- 
Esta mensagem foi verificada pelo sistema de antiv�rus e
 acredita-se estar livre de perigo.



[obm-l] Dúvida num Enunciado

2018-04-25 Por tôpico Luiz Antonio Rodrigues
Olá, pessoal!
Boa tarde!
Estou tentando fazer o exercício abaixo, mas o problema é que eu não
entendi o enunciado...

Determine para quais valores de r (r>0) a implicação é verdadeira:
|x+3| x^2 - 10x + 9 > 0 para todo x real.

Agradeço qualquer ajuda!
Um abraço!
Luiz

-- 
Esta mensagem foi verificada pelo sistema de antiv�rus e
 acredita-se estar livre de perigo.



[obm-l] Re: [obm-l] Dúvida combinatória

2017-11-14 Por tôpico Esdras Muniz
Obs:

$$S(n,\,k_1,\cdots ,k_n)=\frac{n!}{(k_1!\cdots k_n!)(1!)^{k_1}\cdots
(n!)^{k_n}}$$

-- 
Esta mensagem foi verificada pelo sistema de antiv�rus e
 acredita-se estar livre de perigo.



[obm-l] Re: [obm-l] Dúvida combinatória

2017-11-14 Por tôpico Esdras Muniz
Bem, imagine que vc tem [image: n] bolas iguais e quer distribuí-las em
caixas de tamanhos [image: k_1,\,k_2,\,\cdots,k_n], onde na caixa [image:
k_i] cabe [image: i] bolas, e você quer que no final cada caixa esteja
totalmente cheia ou vazia. Isso é equivalente ao problema que você propõe,
e a resposta é:

[image: S(n,\,k_1,k_2,\cdots,k_t)=\frac{n!}{(k_1!\cdots
k_t!)(1!)^{k_1}\cdots(n!)^{k_n}}]

.

Em 13 de novembro de 2017 23:30, Eduardo Henrique 
escreveu:

> Pessoal, estava estudando o seguinte tipo de problema:
>
> Quantas são as soluções inteiras positivas de a+b+c=r, com r inteiro
> positivo. Até aqui ok. A dúvida veio depois:
>
> Quantas são as solução inteiras positivas de 1a+2b+3c=r? E mais
> geralmente, de 1k_1+...+n_kn=r? Alguém sabe como abordar esse tipo de
> problema ou então saberia me indicar um material de estudos?
>
> Obrigado.
>
> Eduardo
>
> --
> Esta mensagem foi verificada pelo sistema de antivírus e
> acredita-se estar livre de perigo.
>



-- 
Esdras Muniz Mota
Mestrando em Matemática
Universidade Federal do Ceará

-- 
Esta mensagem foi verificada pelo sistema de antiv�rus e
 acredita-se estar livre de perigo.



[obm-l] Dúvida combinatória

2017-11-13 Por tôpico Eduardo Henrique
Pessoal, estava estudando o seguinte tipo de problema:

Quantas são as soluções inteiras positivas de a+b+c=r, com r inteiro positivo. 
Até aqui ok. A dúvida veio depois:

Quantas são as solução inteiras positivas de 1a+2b+3c=r? E mais geralmente, de 
1k_1+...+n_kn=r? Alguém sabe como abordar esse tipo de problema ou então 
saberia me indicar um material de estudos?

Obrigado.

Eduardo

-- 
Esta mensagem foi verificada pelo sistema de antivírus e
 acredita-se estar livre de perigo.



[obm-l] Re: [obm-l] Dúvida em como resolver um sistema de eq lineares em N

2017-09-07 Por tôpico Bernardo Freitas Paulo da Costa
2017-09-07 3:33 GMT-03:00 Michel Torres :
> Olá Pessoal
>
> Como ataco esse problema?
>
> possuo duas equções lineares
>
> eq1 = aX+bY+cz+dW
>
> eq2 = eX+fY+gZ+hW
>
> a,b,c,d,e,f,g,h valores conhecidos
>
> Restrição X+Y+Z+W = 6 e X,Y,Z,W assumem somente valores inteiros
> {0,1,2,3,4,5,6}
>
> Quero encontrar para quais valores de X,Y,Z,W  obtenho um valor máximo para
> eq1 e ao mesmo tempo um valor mínimo para eq2

Bom, se você nunca estudou programação linear, esta seria a primeira
parte.  O seu problema parece um PL, com variáveis inteiras.  Eu digo
"parece" porque você deseja ao mesmo tempo maximizar eq1 e minimizar
eq2, como você disse.  E isso, até onde eu entendo, é um problema mal
posto.  Vou tentar dar um exemplo completo:

Suponha que e>f>g>h>0; nestas condições, (0,0,0,6) minimiza eq2, e é o
único que realiza o mínimo
Suponha que (a,b,c,d) = (2,2,1,1); de novo, observe que (X,Y,Z,W) =
(2,2,1,1) maximiza a eq1 e também é o único.

Como fazer para escolher entre estes valores?  Percebe que parece que
"falta um critério de desempate"?  Talvez você queira um valor que
maximiza eq1, e dentre todos os possíveis, um que minimize eq2.  (Isso
daria a solução (2,2,1,1)).  Talvez seja o contrário: é mais
importante minimizar eq2, e você dentre todos que dão o mesmo mínimo,
você escolhe o que maximiza eq1.  E poderia ser outra coisa também...
(só para te dar mais coisas para pesquisar, esta abordagem é conhecida
como "otimização multi-objetivo", e um dos primeiros tópicos é sobre
como transformar isso num problema bem-posto, com solução clara, etc)

Abraços,
-- 
Bernardo Freitas Paulo da Costa

-- 
Esta mensagem foi verificada pelo sistema de antiv�rus e
 acredita-se estar livre de perigo.


=
Instru��es para entrar na lista, sair da lista e usar a lista em
http://www.mat.puc-rio.br/~obmlistas/obm-l.html
=


[obm-l] Dúvida em como resolver um sistema de eq lineares em N

2017-09-07 Por tôpico Michel Torres
Olá Pessoal

Como ataco esse problema?

possuo duas equções lineares

eq1 = aX+bY+cz+dW

eq2 = eX+fY+gZ+hW

a,b,c,d,e,f,g,h valores conhecidos

Restrição X+Y+Z+W = 6 e X,Y,Z,W assumem somente valores inteiros
{0,1,2,3,4,5,6}

Quero encontrar para quais valores de X,Y,Z,W  obtenho um valor máximo para
eq1 e ao mesmo tempo um valor mínimo para eq2


Obrigado pela ajuda

-- 
Esta mensagem foi verificada pelo sistema de antiv�rus e
 acredita-se estar livre de perigo.



[obm-l] Re: [obm-l] Re: [obm-l] Re: [obm-l] Re: [obm-l] Dúvida em uma solução (conjunto denso)

2017-07-10 Por tôpico Pedro Soares
Sim, é uma prova por absurdo.

''...o autor parte de uma hipótese contrária ao resultado pra chegar num
absurdo...''

2017-07-11 1:03 GMT-03:00 Bernardo Freitas Paulo da Costa <
bernardo...@gmail.com>:

> 2017-07-10 18:56 GMT+03:00 Antonio Carlos :
> > Entendi. Muito obrigado, Pedro!
>
> Tem um problema muito sério, que os logs são diferentes...
>
> log_2 3 = log(3)/log(2) = 1.5849625007211563
> log_3 6 = log(6)/log(3) = 1.6309297535714573
>
> Mas o problema está, provavelmente, na primeira hipótese (que ela
> também é falsa).  A demonstração por densidade está certa, e talvez
> seja no meio de um raciocínio por absurdo, mas sei lá...
> --
> Bernardo Freitas Paulo da Costa
>
> --
> Esta mensagem foi verificada pelo sistema de antivírus e
>  acredita-se estar livre de perigo.
>
>
> =
> Instru�ões para entrar na lista, sair da lista e usar a lista em
> http://www.mat.puc-rio.br/~obmlistas/obm-l.html
> =
>

-- 
Esta mensagem foi verificada pelo sistema de antiv�rus e
 acredita-se estar livre de perigo.



[obm-l] Re: [obm-l] Re: [obm-l] Re: [obm-l] Dúvida em uma solução (conjunto denso)

2017-07-10 Por tôpico Bernardo Freitas Paulo da Costa
2017-07-10 18:56 GMT+03:00 Antonio Carlos :
> Entendi. Muito obrigado, Pedro!

Tem um problema muito sério, que os logs são diferentes...

log_2 3 = log(3)/log(2) = 1.5849625007211563
log_3 6 = log(6)/log(3) = 1.6309297535714573

Mas o problema está, provavelmente, na primeira hipótese (que ela
também é falsa).  A demonstração por densidade está certa, e talvez
seja no meio de um raciocínio por absurdo, mas sei lá...
-- 
Bernardo Freitas Paulo da Costa

-- 
Esta mensagem foi verificada pelo sistema de antiv�rus e
 acredita-se estar livre de perigo.


=
Instru��es para entrar na lista, sair da lista e usar a lista em
http://www.mat.puc-rio.br/~obmlistas/obm-l.html
=


[obm-l] Re: [obm-l] Re: [obm-l] Dúvida em uma solução (conjunto denso)

2017-07-10 Por tôpico Antonio Carlos
Entendi. Muito obrigado, Pedro!

On Jul 10, 2017 06:26, "Pedro Soares"  wrote:

> u/v < log_2 3 => u/v < log_3 6 , logo ou log_2 3 é menor ou igual a log_3
> 6 ou o intervalo [log_3 6, log_2 3] não possui nenhum número racional.
>
> u/v < log_3 6 => u/v < log_2 3 , logo ou log_3 6 é menor ou igual a log_2
> 3 ou o intervalo [log_2 3, log_3 6] não possui nenhum número racional.
>
> Como os racionais são densos na reta temos que log_2 3 >= log_3 6 e log_3
> 6 >= log_2 3 ==> log_2 3 = log_3 6, o que é falso. Ou isso ou os intervalos
> seriam degenerados o que também implicaria em log_2 3 = log_3 6.
> Assim, vc chega em um absurdo.
>
> Sacou?
>
>
>
> 2017-07-09 17:03 GMT-03:00 Antonio Carlos :
>
>> Oi pessoal,
>>
>> Estava lendo uma resolução de uma questão, e em uma passagem se chega à
>> seguinte implicação (u e v são naturais, log_a x é o logaritmo de x na base
>> a):
>>
>>  u/v < log_2 3 se e somente se u/v < log_3 6, e como os racionais são
>> densos, temos que a equivalência acima implica que log_2 3 = log_3 6.
>>
>> Tudo bem com a equivalência, o autor parte de uma hipótese contrária ao
>> resultado pra chegar num absurdo, o que não entendi foi a implicação usando
>> que Q é denso. Eu já fiz um curso de análise e tenho alguma noção do que é
>> um conjunto ser denso. Se alguém puder me ajudar a entender a passagem eu
>> agradeço.
>>
>> Att,
>> Antonio
>>
>>
>>
>> --
>> Esta mensagem foi verificada pelo sistema de antivírus e
>> acredita-se estar livre de perigo.
>
>
>
> --
> Esta mensagem foi verificada pelo sistema de antivírus e
> acredita-se estar livre de perigo.

-- 
Esta mensagem foi verificada pelo sistema de antiv�rus e
 acredita-se estar livre de perigo.



[obm-l] Re: [obm-l] Dúvida em uma solução (conjunto denso)

2017-07-10 Por tôpico Pedro Soares
u/v < log_2 3 => u/v < log_3 6 , logo ou log_2 3 é menor ou igual a log_3 6
ou o intervalo [log_3 6, log_2 3] não possui nenhum número racional.

u/v < log_3 6 => u/v < log_2 3 , logo ou log_3 6 é menor ou igual a log_2 3
ou o intervalo [log_2 3, log_3 6] não possui nenhum número racional.

Como os racionais são densos na reta temos que log_2 3 >= log_3 6 e log_3 6
>= log_2 3 ==> log_2 3 = log_3 6, o que é falso. Ou isso ou os intervalos
seriam degenerados o que também implicaria em log_2 3 = log_3 6.
Assim, vc chega em um absurdo.

Sacou?



2017-07-09 17:03 GMT-03:00 Antonio Carlos :

> Oi pessoal,
>
> Estava lendo uma resolução de uma questão, e em uma passagem se chega à
> seguinte implicação (u e v são naturais, log_a x é o logaritmo de x na base
> a):
>
>  u/v < log_2 3 se e somente se u/v < log_3 6, e como os racionais são
> densos, temos que a equivalência acima implica que log_2 3 = log_3 6.
>
> Tudo bem com a equivalência, o autor parte de uma hipótese contrária ao
> resultado pra chegar num absurdo, o que não entendi foi a implicação usando
> que Q é denso. Eu já fiz um curso de análise e tenho alguma noção do que é
> um conjunto ser denso. Se alguém puder me ajudar a entender a passagem eu
> agradeço.
>
> Att,
> Antonio
>
>
>
> --
> Esta mensagem foi verificada pelo sistema de antivírus e
> acredita-se estar livre de perigo.

-- 
Esta mensagem foi verificada pelo sistema de antiv�rus e
 acredita-se estar livre de perigo.



[obm-l] Dúvida em uma solução (conjunto denso)

2017-07-09 Por tôpico Antonio Carlos
Oi pessoal,

Estava lendo uma resolução de uma questão, e em uma passagem se chega à
seguinte implicação (u e v são naturais, log_a x é o logaritmo de x na base
a):

 u/v < log_2 3 se e somente se u/v < log_3 6, e como os racionais são
densos, temos que a equivalência acima implica que log_2 3 = log_3 6.

Tudo bem com a equivalência, o autor parte de uma hipótese contrária ao
resultado pra chegar num absurdo, o que não entendi foi a implicação usando
que Q é denso. Eu já fiz um curso de análise e tenho alguma noção do que é
um conjunto ser denso. Se alguém puder me ajudar a entender a passagem eu
agradeço.

Att,
Antonio

-- 
Esta mensagem foi verificada pelo sistema de antiv�rus e
 acredita-se estar livre de perigo.



[obm-l] Re: [obm-l] Re: [obm-l] Re: [obm-l] Re: [obm-l] Dúvida sobre a Obm U

2016-07-26 Por tôpico Carlos Gomes
Um bom livro é Razvan Gelca, Titu Andreescu-Putnam and Beyond (2007)

Cgomes.

Em 26 de julho de 2016 08:57, Otávio Araújo 
escreveu:

> Não, onde posso conseguir? e do que ela trata?
>
> Em 25 de julho de 2016 11:32, Carlos Victor 
> escreveu:
>
>>
>>
>>
>> Oi Otávio,
>>
>> Você já viu a Revista Matemática Universitária da SBM ?
>>
>> Em 25/07/2016 10:09, Otávio Araújo escreveu:
>>
>>
>>
>> Pois é, se algum professor com experiência em olimpíadas, como o Nicolau
>> por exemplo, respondesse minha pergunta seria de grande ajuda
>>
>> Em 24 de jul de 2016, às 23:25, Israel Meireles Chrisostomo <
>> israelmchrisost...@gmail.com> escreveu:
>>
>> Boa pergunta, eu também tenho interesse em participar da OBM U e
>> gostaria de umas dicas
>>
>> Em 16 de julho de 2016 13:29, Otávio Araújo 
>> escreveu:
>>
>>> Galera, gostaria que vocês me dessem dicas de o que estudar, como
>>> estudar e por quais livros e materiais estudar para a prova da Obm nível
>>> universitário...
>>> Estou muito interessado em participar, mas fico meio confuso por onde
>>> estudar...
>>> Por favor me ajudem
>>> --
>>> Esta mensagem foi verificada pelo sistema de antivírus e
>>> Â acredita-se estar livre de perigo.
>>>
>>>
>>> =
>>> Instruções para entrar na lista, sair da lista e usar a lista em
>>> http://www.mat.puc-rio.br/~obmlistas/obm-l.html
>>> =
>>>
>>
>> --
>> Esta mensagem foi verificada pelo sistema de antivírus e
>> acredita-se estar livre de perigo.
>>
>>
>> --
>> Esta mensagem foi verificada pelo sistema de antivrus e
>> acredita-se estar livre de perigo.
>>
>>
>> --
>> Esta mensagem foi verificada pelo sistema de antivírus e
>> acredita-se estar livre de perigo.
>>
>
>
> --
> Esta mensagem foi verificada pelo sistema de antivírus e
> acredita-se estar livre de perigo.
>

-- 
Esta mensagem foi verificada pelo sistema de antiv�rus e
 acredita-se estar livre de perigo.



[obm-l] Re: [obm-l] Re: [obm-l] Re: [obm-l] Dúvida sobre a Obm U

2016-07-26 Por tôpico Otávio Araújo
Não, onde posso conseguir? e do que ela trata?

Em 25 de julho de 2016 11:32, Carlos Victor 
escreveu:

>
>
>
> Oi Otávio,
>
> Você já viu a Revista Matemática Universitária da SBM ?
>
> Em 25/07/2016 10:09, Otávio Araújo escreveu:
>
>
>
> Pois é, se algum professor com experiência em olimpíadas, como o Nicolau
> por exemplo, respondesse minha pergunta seria de grande ajuda
>
> Em 24 de jul de 2016, às 23:25, Israel Meireles Chrisostomo <
> israelmchrisost...@gmail.com> escreveu:
>
> Boa pergunta, eu também tenho interesse em participar da OBM U e gostaria
> de umas dicas
>
> Em 16 de julho de 2016 13:29, Otávio Araújo 
> escreveu:
>
>> Galera, gostaria que vocês me dessem dicas de o que estudar, como
>> estudar e por quais livros e materiais estudar para a prova da Obm nível
>> universitário...
>> Estou muito interessado em participar, mas fico meio confuso por onde
>> estudar...
>> Por favor me ajudem
>> --
>> Esta mensagem foi verificada pelo sistema de antivírus e
>> Â acredita-se estar livre de perigo.
>>
>>
>> =
>> Instruções para entrar na lista, sair da lista e usar a lista em
>> http://www.mat.puc-rio.br/~obmlistas/obm-l.html
>> =
>>
>
> --
> Esta mensagem foi verificada pelo sistema de antivírus e
> acredita-se estar livre de perigo.
>
>
> --
> Esta mensagem foi verificada pelo sistema de antivrus e
> acredita-se estar livre de perigo.
>
>
> --
> Esta mensagem foi verificada pelo sistema de antivírus e
> acredita-se estar livre de perigo.
>

-- 
Esta mensagem foi verificada pelo sistema de antiv�rus e
 acredita-se estar livre de perigo.



[obm-l] Re: [obm-l] Re: [obm-l] Dúvida sobre a Obm U

2016-07-25 Por tôpico Carlos Victor
 

Oi Otávio, 

Você já viu a Revista Matemática Universitária da SBM ? 

Em 25/07/2016 10:09, Otávio Araújo escreveu: 

> Pois é, se algum professor com experiência em olimpíadas, como o Nicolau por 
> exemplo, respondesse minha pergunta seria de grande ajuda 
> 
> Em 24 de jul de 2016, às 23:25, Israel Meireles Chrisostomo 
>  escreveu:
> 
> Boa pergunta, eu tambÃ(c)m tenho interesse em participar da OBM U e gostaria 
> de umas dicas 
> 
> Em 16 de julho de 2016 13:29, Otávio Araújo  
> escreveu:
> Galera, gostaria que vocês me dessem dicas de o que estudar, como estudar 
> e por quais livros e materiais estudar para a prova da Obm nível 
> universitário...
> Estou muito interessado em participar, mas fico meio confuso por onde 
> estudar...
> Por favor me ajudem
> --
> Esta mensagem foi verificada pelo sistema de antivírus e
> Â acredita-se estar livre de perigo.
> 
> =
> Instruções para entrar na lista, sair da lista e usar a lista em
> http://www.mat.puc-rio.br/~obmlistas/obm-l.html [1]
> =
> 
> -- 
> Esta mensagem foi verificada pelo sistema de antivírus e 
> acredita-se estar livre de perigo.

-- 
Esta mensagem foi verificada pelo sistema de antivrus e 
acredita-se estar livre de perigo. 

Links:
--
[1] http://www.mat.puc-rio.br/~obmlistas/obm-l.html

-- 
Esta mensagem foi verificada pelo sistema de antiv�rus e
 acredita-se estar livre de perigo.



[obm-l] Re: [obm-l] Re: [obm-l] Re: [obm-l] Re: [obm-l] Dúvida sobre a Obm U

2016-07-25 Por tôpico Tiago Sandino
Égua ma, sou mais ou menos da UFC, de qualquer forma, começar matemática
UFC prox ano. Fiz olimpíada um tempo, imergi totalmente nisso. Fiz e
trabalhei com engenharia elétrica uns anos, larguei o curso no final pq o
negócio na engenharia era próprio e precisava de tempo. Atualmente tô dando
aula de turma ITA IME e olimpíada de mat no Colégio Militar. Não sei se eu
vou poder fazer a prova da OBMU, mas estudarei. E a olimpíada universitária
é uma continuação da não universitária, daí meu interesse, já que sou
professor disso.
Precisando de dica, ta aí meu contato. 85 9 99134896. Se tem uma coisa que
eu sei sobre olimpíada é que o cara tem que tá no meio, tem que falar sobre
isso, tem que conhecer pessoas do meio etc. Só assim vc evolui. Só assim vc
passa de ser um cara que consegue aplicar fórmula a ser um cara que "cria"
matemática todo dia, que na minha concepção é a maior realização pessoal na
olimpíada.


Em 25 de julho de 2016 17:19, Otávio Araújo 
escreveu:

>
>
> Égua Tiago, eu também sou do Ceará mas meu celular atualmente não tem
> chip Mas tu é da UFC Tiago? E ainda estou esperando algum professor com
> experiência em olimpíadas de matemática responder a minha pergunta
>
> Em 25 de jul de 2016, às 13:38, Tiago Sandino 
> escreveu:
>
> Oi pessoal.
> Tem diversos livros de olimpíadas para graduandos (undergrads) ou com
> capítulos de temas exclusivamente (até onde eu saiba) universitários.
> Grátis na net, que eu saiba, tem muita coisa no AOPS. Dois links aqui:
> 1) *Fórum*: https://www.artofproblemsolving.com/community/c7_college_math
> 2) *Fórum por Competições*:
> https://www.artofproblemsolving.com/community/c15_undergraduate_contests
>
> Sou do Ceará, tava meio afastado da Matemática, mas fiz as pazes com ela
> recentemente. Se alguém quiser formar um grupo de estudo pelo Whatsapp...
> segue meu número: 85 9 9913 4896.
>
> Att.
> Tiago Sandino
>
> Em 25 de julho de 2016 10:20, Raul Alves  escreveu:
>
>> Também tenho interesse na OBMU, e a 1ª fase tá chegando.
>> Se algum professor puder organizar algum material de apoio, seria de
>> grande ajuda
>>
>> Em 25 de julho de 2016 10:09, Otávio Araújo 
>> escreveu:
>>
>>>
>>>
>>> Pois é, se algum professor com experiência em olimpíadas, como o
>>> Nicolau por exemplo, respondesse minha pergunta seria de grande ajuda
>>>
>>> Em 24 de jul de 2016, Ã s 23:25, Israel Meireles Chrisostomo <
>>> israelmchrisost...@gmail.com> escreveu:
>>>
>>> Boa pergunta, eu também tenho interesse em participar da OBM U e
>>> gostaria de umas dicas
>>>
>>> Em 16 de julho de 2016 13:29, Otávio Araújo <
>>> otavio17.ara...@gmail.com> escreveu:
>>>
 Galera, gostaria que vocês me dessem dicas de o que estudar,
 como estudar e por quais livros e materiais estudar para a prova da Obm
 nível universitário...
 Estou muito interessado em participar, mas fico meio confuso por onde
 estudar...
 Por favor me ajudem
 --
 Esta mensagem foi verificada pelo sistema de antivírus e
  acredita-se estar livre de perigo.



 =
 Instruções para entrar na lista, sair da lista e usar a lista em
 http://www.mat.puc-rio.br/~obmlistas/obm-l.html

 =

>>>
>>>
>>> --
>>> Esta mensagem foi verificada pelo sistema de antivírus e
>>> acredita-se estar livre de perigo.
>>>
>>>
>>> --
>>> Esta mensagem foi verificada pelo sistema de antivírus e
>>> acredita-se estar livre de perigo.
>>>
>>
>>
>>
>> --
>> *Raul Lima Alves*
>>
>> *Estagiário na Aton Engenharia*
>> *Estudante de Engenharia de Computação - UFBA*
>> *Telefone: (71) 9103-0878*
>> *Facebook:Â *https://www.facebook.com/raul.alves.161
>> *LinkedIn*:Â https://br.linkedin.com/in/raul-alves-8b090228
>> 
>>
>>
>>
>> --
>> Esta mensagem foi verificada pelo sistema de antivírus e
>> acredita-se estar livre de perigo.
>>
>
>
> --
> Esta mensagem foi verificada pelo sistema de antivírus e
> acredita-se estar livre de perigo.
>
>
> --
> Esta mensagem foi verificada pelo sistema de antivírus e
> acredita-se estar livre de perigo.
>

-- 
Esta mensagem foi verificada pelo sistema de antiv�rus e
 acredita-se estar livre de perigo.



Re: [obm-l] Re: [obm-l] Re: [obm-l] Re: [obm-l] Dúvida sobre a Obm U

2016-07-25 Por tôpico Otávio Araújo


Égua Tiago, eu também sou do Ceará mas meu celular atualmente não tem chip 
Mas tu é da UFC Tiago? E ainda estou esperando algum professor com experiência 
em olimpíadas de matemática responder a minha pergunta 

> Em 25 de jul de 2016, às 13:38, Tiago Sandino  
> escreveu:
> 
> Oi pessoal.
> Tem diversos livros de olimpíadas para graduandos (undergrads) ou com 
> capítulos de temas exclusivamente (até onde eu saiba) universitários. 
> Grátis na net, que eu saiba, tem muita coisa no AOPS. Dois links aqui:
> 1) Fórum: https://www.artofproblemsolving.com/community/c7_college_math
> 2) Fórum por Competições: 
> https://www.artofproblemsolving.com/community/c15_undergraduate_contests
> 
> Sou do Ceará, tava meio afastado da Matemática, mas fiz as pazes com ela 
> recentemente. Se alguém quiser formar um grupo de estudo pelo Whatsapp... 
> segue meu número: 85 9 9913 4896.
> 
> Att.
> Tiago Sandino
> 
> Em 25 de julho de 2016 10:20, Raul Alves  escreveu:
>> Também tenho interesse na OBMU, e a 1ª fase tá chegando.
>> Se algum professor puder organizar algum material de apoio, seria de grande 
>> ajuda
>> 
>> Em 25 de julho de 2016 10:09, Otávio Araújo  
>> escreveu:
>>> 
>>> 
>>> Pois é, se algum professor com experiência em olimpíadas, como o Nicolau 
>>> por exemplo, respondesse minha pergunta seria de grande ajuda
>>> 
>>> Em 24 de jul de 2016, Ã s 23:25, Israel Meireles Chrisostomo 
>>>  escreveu:
>>> 
 Boa pergunta, eu também tenho interesse em participar da OBM U e 
 gostaria de umas dicas
 
 Em 16 de julho de 2016 13:29, Otávio Araújo 
  escreveu:
> Galera, gostaria que vocês me dessem dicas de o que estudar, como 
> estudar e por quais livros e materiais estudar para a prova da Obm 
> nível universitário...
> Estou muito interessado em participar, mas fico meio confuso por onde 
> estudar...
> Por favor me ajudem
> --
> Esta mensagem foi verificada pelo sistema de antivírus e
>  acredita-se estar livre de perigo.
> 
> 
> =
> Instruções para entrar na lista, sair da lista e usar a lista em
> http://www.mat.puc-rio.br/~obmlistas/obm-l.html
> =
 
 
 -- 
 Esta mensagem foi verificada pelo sistema de antivírus e 
 acredita-se estar livre de perigo.
>>> 
>>> -- 
>>> Esta mensagem foi verificada pelo sistema de antivírus e 
>>> acredita-se estar livre de perigo.
>> 
>> 
>> 
>> -- 
>> Raul Lima Alves
>> 
>> Estagiário na Aton Engenharia
>> Estudante de Engenharia de Computação - UFBA
>> Telefone: (71) 9103-0878
>> Facebook:Â https://www.facebook.com/raul.alves.161
>> LinkedIn:Â https://br.linkedin.com/in/raul-alves-8b090228
>> 
>> 
>> 
>> -- 
>> Esta mensagem foi verificada pelo sistema de antivírus e 
>> acredita-se estar livre de perigo.
> 
> 
> -- 
> Esta mensagem foi verificada pelo sistema de antivírus e 
> acredita-se estar livre de perigo.

-- 
Esta mensagem foi verificada pelo sistema de antiv�rus e
 acredita-se estar livre de perigo.



[obm-l] Re: [obm-l] Dúvida sobre a Obm U

2016-07-25 Por tôpico Jeferson Almir
Boa iniciativa Sandino!!
Um prova que se aproxima bastante é a do Putnam ( universitária americana )
e o livro Putnam and Beyond seria um bom começo.
Refazer prova passadas da OBMU  e depois ver a solução  possíveis dúvidas é
um ótimo começo. Existe também a universitária colombiana que que a parte
de álgebra linear eu particularmente acho ótimo. Abraço

Em segunda-feira, 25 de julho de 2016, Tiago Sandino <
tiagosandi...@gmail.com> escreveu:

> Oi pessoal.
> Tem diversos livros de olimpíadas para graduandos (undergrads) ou com
> capítulos de temas exclusivamente (até onde eu saiba) universitários.
> Grátis na net, que eu saiba, tem muita coisa no AOPS. Dois links aqui:
> 1) *Fórum*: https://www.artofproblemsolving.com/community/c7_college_math
> 2) *Fórum por Competições*:
> https://www.artofproblemsolving.com/community/c15_undergraduate_contests
>
> Sou do Ceará, tava meio afastado da Matemática, mas fiz as pazes com ela
> recentemente. Se alguém quiser formar um grupo de estudo pelo Whatsapp...
> segue meu número: 85 9 9913 4896.
>
> Att.
> Tiago Sandino
>
> Em 25 de julho de 2016 10:20, Raul Alves  > escreveu:
>
>> Também tenho interesse na OBMU, e a 1ª fase tá chegando.
>> Se algum professor puder organizar algum material de apoio, seria de
>> grande ajuda
>>
>> Em 25 de julho de 2016 10:09, Otávio Araújo > > escreveu:
>>
>>>
>>>
>>> Pois é, se algum professor com experiência em olimpíadas, como o Nicolau
>>> por exemplo, respondesse minha pergunta seria de grande ajuda
>>>
>>> Em 24 de jul de 2016, às 23:25, Israel Meireles Chrisostomo <
>>> israelmchrisost...@gmail.com
>>> >
>>> escreveu:
>>>
>>> Boa pergunta, eu também tenho interesse em participar da OBM U e
>>> gostaria de umas dicas
>>>
>>> Em 16 de julho de 2016 13:29, Otávio Araújo >> > escreveu:
>>>
 Galera, gostaria que vocês me dessem dicas de o que estudar, como
 estudar e por quais livros e materiais estudar para a prova da Obm nível
 universitário...
 Estou muito interessado em participar, mas fico meio confuso por onde
 estudar...
 Por favor me ajudem
 --
 Esta mensagem foi verificada pelo sistema de antivírus e
 Â acredita-se estar livre de perigo.



 =
 Instruções para entrar na lista, sair da lista e usar a lista em
 http://www.mat.puc-rio.br/~obmlistas/obm-l.html

 =

>>>
>>>
>>> --
>>> Esta mensagem foi verificada pelo sistema de antivírus e
>>> acredita-se estar livre de perigo.
>>>
>>>
>>> --
>>> Esta mensagem foi verificada pelo sistema de antivírus e
>>> acredita-se estar livre de perigo.
>>>
>>
>>
>>
>> --
>> *Raul Lima Alves*
>>
>> *Estagiário na Aton Engenharia*
>> *Estudante de Engenharia de Computação - UFBA*
>> *Telefone: (71) 9103-0878*
>> *Facebook: *https://www.facebook.com/raul.alves.161
>> *LinkedIn*: https://br.linkedin.com/in/raul-alves-8b090228
>> 
>>
>>
>>
>> --
>> Esta mensagem foi verificada pelo sistema de antivírus e
>> acredita-se estar livre de perigo.
>>
>
>
> --
> Esta mensagem foi verificada pelo sistema de antivírus e
> acredita-se estar livre de perigo.

-- 
Esta mensagem foi verificada pelo sistema de antiv�rus e
 acredita-se estar livre de perigo.



[obm-l] Re: [obm-l] Re: [obm-l] Re: [obm-l] Dúvida sobre a Obm U

2016-07-25 Por tôpico Tiago Sandino
Oi pessoal.
Tem diversos livros de olimpíadas para graduandos (undergrads) ou com
capítulos de temas exclusivamente (até onde eu saiba) universitários.
Grátis na net, que eu saiba, tem muita coisa no AOPS. Dois links aqui:
1) *Fórum*: https://www.artofproblemsolving.com/community/c7_college_math
2) *Fórum por Competições*:
https://www.artofproblemsolving.com/community/c15_undergraduate_contests

Sou do Ceará, tava meio afastado da Matemática, mas fiz as pazes com ela
recentemente. Se alguém quiser formar um grupo de estudo pelo Whatsapp...
segue meu número: 85 9 9913 4896.

Att.
Tiago Sandino

Em 25 de julho de 2016 10:20, Raul Alves  escreveu:

> Também tenho interesse na OBMU, e a 1ª fase tá chegando.
> Se algum professor puder organizar algum material de apoio, seria de
> grande ajuda
>
> Em 25 de julho de 2016 10:09, Otávio Araújo 
> escreveu:
>
>>
>>
>> Pois é, se algum professor com experiência em olimpíadas, como o Nicolau
>> por exemplo, respondesse minha pergunta seria de grande ajuda
>>
>> Em 24 de jul de 2016, às 23:25, Israel Meireles Chrisostomo <
>> israelmchrisost...@gmail.com> escreveu:
>>
>> Boa pergunta, eu também tenho interesse em participar da OBM U e
>> gostaria de umas dicas
>>
>> Em 16 de julho de 2016 13:29, Otávio Araújo 
>> escreveu:
>>
>>> Galera, gostaria que vocês me dessem dicas de o que estudar, como
>>> estudar e por quais livros e materiais estudar para a prova da Obm nível
>>> universitário...
>>> Estou muito interessado em participar, mas fico meio confuso por onde
>>> estudar...
>>> Por favor me ajudem
>>> --
>>> Esta mensagem foi verificada pelo sistema de antivírus e
>>> Â acredita-se estar livre de perigo.
>>>
>>>
>>> =
>>> Instruções para entrar na lista, sair da lista e usar a lista em
>>> http://www.mat.puc-rio.br/~obmlistas/obm-l.html
>>> =
>>>
>>
>>
>> --
>> Esta mensagem foi verificada pelo sistema de antivírus e
>> acredita-se estar livre de perigo.
>>
>>
>> --
>> Esta mensagem foi verificada pelo sistema de antivírus e
>> acredita-se estar livre de perigo.
>>
>
>
>
> --
> *Raul Lima Alves*
>
> *Estagiário na Aton Engenharia*
> *Estudante de Engenharia de Computação - UFBA*
> *Telefone: (71) 9103-0878*
> *Facebook: *https://www.facebook.com/raul.alves.161
> *LinkedIn*: https://br.linkedin.com/in/raul-alves-8b090228
> 
>
>
>
> --
> Esta mensagem foi verificada pelo sistema de antivírus e
> acredita-se estar livre de perigo.
>

-- 
Esta mensagem foi verificada pelo sistema de antiv�rus e
 acredita-se estar livre de perigo.



[obm-l] Re: [obm-l] Re: [obm-l] Dúvida sobre a Obm U

2016-07-25 Por tôpico Raul Alves
Também tenho interesse na OBMU, e a 1ª fase tá chegando.
Se algum professor puder organizar algum material de apoio, seria de grande
ajuda

Em 25 de julho de 2016 10:09, Otávio Araújo 
escreveu:

>
>
> Pois é, se algum professor com experiência em olimpíadas, como o Nicolau
> por exemplo, respondesse minha pergunta seria de grande ajuda
>
> Em 24 de jul de 2016, às 23:25, Israel Meireles Chrisostomo <
> israelmchrisost...@gmail.com> escreveu:
>
> Boa pergunta, eu também tenho interesse em participar da OBM U e gostaria
> de umas dicas
>
> Em 16 de julho de 2016 13:29, Otávio Araújo 
> escreveu:
>
>> Galera, gostaria que vocês me dessem dicas de o que estudar, como
>> estudar e por quais livros e materiais estudar para a prova da Obm nível
>> universitário...
>> Estou muito interessado em participar, mas fico meio confuso por onde
>> estudar...
>> Por favor me ajudem
>> --
>> Esta mensagem foi verificada pelo sistema de antivírus e
>> Â acredita-se estar livre de perigo.
>>
>>
>> =
>> Instruções para entrar na lista, sair da lista e usar a lista em
>> http://www.mat.puc-rio.br/~obmlistas/obm-l.html
>> =
>>
>
>
> --
> Esta mensagem foi verificada pelo sistema de antivírus e
> acredita-se estar livre de perigo.
>
>
> --
> Esta mensagem foi verificada pelo sistema de antivírus e
> acredita-se estar livre de perigo.
>



-- 
*Raul Lima Alves*

*Estagiário na Aton Engenharia*
*Estudante de Engenharia de Computação - UFBA*
*Telefone: (71) 9103-0878*
*Facebook: *https://www.facebook.com/raul.alves.161
*LinkedIn*: https://br.linkedin.com/in/raul-alves-8b090228


-- 
Esta mensagem foi verificada pelo sistema de antiv�rus e
 acredita-se estar livre de perigo.



Re: [obm-l] Re: [obm-l] Dúvida sobre a Obm U

2016-07-25 Por tôpico Otávio Araújo


Pois é, se algum professor com experiência em olimpíadas, como o Nicolau por 
exemplo, respondesse minha pergunta seria de grande ajuda

> Em 24 de jul de 2016, às 23:25, Israel Meireles Chrisostomo 
>  escreveu:
> 
> Boa pergunta, eu também tenho interesse em participar da OBM U e gostaria de 
> umas dicas
> 
> Em 16 de julho de 2016 13:29, Otávio Araújo  
> escreveu:
>> Galera, gostaria que vocês me dessem dicas de o que estudar, como estudar 
>> e por quais livros e materiais estudar para a prova da Obm nível 
>> universitário...
>> Estou muito interessado em participar, mas fico meio confuso por onde 
>> estudar...
>> Por favor me ajudem
>> --
>> Esta mensagem foi verificada pelo sistema de antivírus e
>> Â acredita-se estar livre de perigo.
>> 
>> 
>> =
>> Instruções para entrar na lista, sair da lista e usar a lista em
>> http://www.mat.puc-rio.br/~obmlistas/obm-l.html
>> =
> 
> 
> -- 
> Esta mensagem foi verificada pelo sistema de antivírus e 
> acredita-se estar livre de perigo.

-- 
Esta mensagem foi verificada pelo sistema de antiv�rus e
 acredita-se estar livre de perigo.



[obm-l] Re: [obm-l] Dúvida sobre a Obm U

2016-07-24 Por tôpico Israel Meireles Chrisostomo
Boa pergunta, eu também tenho interesse em participar da OBM U e gostaria
de umas dicas

Em 16 de julho de 2016 13:29, Otávio Araújo 
escreveu:

> Galera, gostaria que vocês me dessem dicas de o que estudar, como estudar
> e por quais livros e materiais estudar para a prova da Obm nível
> universitário...
> Estou muito interessado em participar, mas fico meio confuso por onde
> estudar...
> Por favor me ajudem
> --
> Esta mensagem foi verificada pelo sistema de antivírus e
>  acredita-se estar livre de perigo.
>
>
> =
> Instruções para entrar na lista, sair da lista e usar a lista em
> http://www.mat.puc-rio.br/~obmlistas/obm-l.html
> =
>

-- 
Esta mensagem foi verificada pelo sistema de antiv�rus e
 acredita-se estar livre de perigo.



[obm-l] Dúvida sobre a Obm U

2016-07-16 Por tôpico Otávio Araújo
Galera, gostaria que vocês me dessem dicas de o que estudar, como estudar e por 
quais livros e materiais estudar para a prova da Obm nível universitário...
Estou muito interessado em participar, mas fico meio confuso por onde estudar...
Por favor me ajudem
-- 
Esta mensagem foi verificada pelo sistema de antiv�rus e
 acredita-se estar livre de perigo.


=
Instru��es para entrar na lista, sair da lista e usar a lista em
http://www.mat.puc-rio.br/~obmlistas/obm-l.html
=


[obm-l] Re: [obm-l] Dúvida em Geometria Espacial

2016-06-08 Por tôpico ilhadepaqueta
Você monta a equação do volume com uma variavel só por exemplo r, tirando o h 
em função do r na equação da area total.
Faz a derivada de V em relação a r e verifica qual o valor de máximo.
Abraços
Hermann

From: Daniel Rocha 
Sent: Wednesday, June 8, 2016 7:17 PM
To: obm-l@mat.puc-rio.br 
Subject: [obm-l] Dúvida em Geometria Espacial

Alguém poderia, por favor, solucionar a questão abaixo:


Uma lata de forma cilíndrica, com tampa, deve ser construída com 60 cm^2 de 
folha de alumínio. Se r é o raio da base e h é a altura da lata que 
proporcionam o volume máximo, então o valor de r/h é:


GABARITO: 1/2

-- 
Esta mensagem foi verificada pelo sistema de antiv�rus e 
acredita-se estar livre de perigo. 
-- 
Esta mensagem foi verificada pelo sistema de antiv�rus e
 acredita-se estar livre de perigo.



[obm-l] Re: [obm-l] Re: [obm-l] Dúvida em Geometria Espacial

2016-06-08 Por tôpico Daniel Rocha
Muito Obrigado, Grande Carlos !!!

Em 8 de junho de 2016 20:13, Carlos Gomes  escreveu:

> Vc pode fazer assim:
>
> área total = 60 ==> 2.pi.r^2+2.pi.r.h=60  ==>h=(60-2.pi.r^2)/(2.pi.r)   (*)
> Por outro lado o volume é
>
> V=pi.r^2.h
>
> substituindo a expressão (*) do h , segue que
>
> V=60r-2.pi.r^3
>
> Fazendo dV/dr=0 (derivada igual a zero para achar os pontos críticos),
> segue que
>
> 0=60-6.pir^2  ==> r=sqtr(10/pi) (é ponto de máximo, pois
> d^2V/dr^2(sqtr(10/pi))<0
>
> substituindo com esse valor de r na expressão (*), segue que
> h=2.sqrt(10/pi).
>
> O que revela que r/h=sqtr(10/pi) / 2.sqtr(10/pi)=1/2.
>
> Cgomes.
>
> Em 8 de junho de 2016 19:17, Daniel Rocha 
> escreveu:
>
>> Alguém poderia, por favor, solucionar a questão abaixo:
>>
>> Uma lata de forma cilíndrica, com tampa, deve ser construída com 60 cm^2
>> de folha de alumínio. Se r é o raio da base e h é a altura da lata que
>> proporcionam o volume máximo, então o valor de r/h é:
>>
>> GABARITO: 1/2
>>
>> --
>> Esta mensagem foi verificada pelo sistema de antivírus e
>> acredita-se estar livre de perigo.
>
>
>
> --
> Esta mensagem foi verificada pelo sistema de antivírus e
> acredita-se estar livre de perigo.

-- 
Esta mensagem foi verificada pelo sistema de antiv�rus e
 acredita-se estar livre de perigo.



[obm-l] Re: [obm-l] Dúvida em Geometria Espacial

2016-06-08 Por tôpico Carlos Gomes
Vc pode fazer assim:

área total = 60 ==> 2.pi.r^2+2.pi.r.h=60  ==>h=(60-2.pi.r^2)/(2.pi.r)   (*)
Por outro lado o volume é

V=pi.r^2.h

substituindo a expressão (*) do h , segue que

V=60r-2.pi.r^3

Fazendo dV/dr=0 (derivada igual a zero para achar os pontos críticos),
segue que

0=60-6.pir^2  ==> r=sqtr(10/pi) (é ponto de máximo, pois
d^2V/dr^2(sqtr(10/pi))<0

substituindo com esse valor de r na expressão (*), segue que
h=2.sqrt(10/pi).

O que revela que r/h=sqtr(10/pi) / 2.sqtr(10/pi)=1/2.

Cgomes.

Em 8 de junho de 2016 19:17, Daniel Rocha 
escreveu:

> Alguém poderia, por favor, solucionar a questão abaixo:
>
> Uma lata de forma cilíndrica, com tampa, deve ser construída com 60 cm^2
> de folha de alumínio. Se r é o raio da base e h é a altura da lata que
> proporcionam o volume máximo, então o valor de r/h é:
>
> GABARITO: 1/2
>
> --
> Esta mensagem foi verificada pelo sistema de antivírus e
> acredita-se estar livre de perigo.

-- 
Esta mensagem foi verificada pelo sistema de antiv�rus e
 acredita-se estar livre de perigo.



[obm-l] Dúvida em Geometria Espacial

2016-06-08 Por tôpico Daniel Rocha
Alguém poderia, por favor, solucionar a questão abaixo:

Uma lata de forma cilíndrica, com tampa, deve ser construída com 60 cm^2 de
folha de alumínio. Se r é o raio da base e h é a altura da lata que
proporcionam o volume máximo, então o valor de r/h é:

GABARITO: 1/2

-- 
Esta mensagem foi verificada pelo sistema de antiv�rus e
 acredita-se estar livre de perigo.



[obm-l] Re: [obm-l] Re: [obm-l] Dúvida em Logaritmos

2016-06-07 Por tôpico Daniel Rocha
Muito Obrigado (mais uma vez), Carlos !!!

Em 6 de junho de 2016 22:02, Carlos Gomes  escreveu:

> log[(sqrt 2)^(x-2)] = x ==>
> (x-2)log(sqrt 2) = x ==>
> x=2log(sqrt2)/(log(sqrt2)-1).
>
> Cgomes.
>
> Em 6 de junho de 2016 19:23, Daniel Rocha 
> escreveu:
>
>> Alguém poderia, por favor, solucionar o problema abaixo:
>>
>> Ache a solução real da equação:
>> log[(sqrt 2)^(x-2)] = x
>>
>> --
>> Esta mensagem foi verificada pelo sistema de antivírus e
>> acredita-se estar livre de perigo.
>
>
>
> --
> Esta mensagem foi verificada pelo sistema de antivírus e
> acredita-se estar livre de perigo.

-- 
Esta mensagem foi verificada pelo sistema de antiv�rus e
 acredita-se estar livre de perigo.



[obm-l] Re: [obm-l] Dúvida em Logaritmos

2016-06-06 Por tôpico Carlos Gomes
log[(sqrt 2)^(x-2)] = x ==>
(x-2)log(sqrt 2) = x ==>
x=2log(sqrt2)/(log(sqrt2)-1).

Cgomes.

Em 6 de junho de 2016 19:23, Daniel Rocha 
escreveu:

> Alguém poderia, por favor, solucionar o problema abaixo:
>
> Ache a solução real da equação:
> log[(sqrt 2)^(x-2)] = x
>
> --
> Esta mensagem foi verificada pelo sistema de antivírus e
> acredita-se estar livre de perigo.

-- 
Esta mensagem foi verificada pelo sistema de antiv�rus e
 acredita-se estar livre de perigo.



[obm-l] Dúvida em Logaritmos

2016-06-06 Por tôpico Daniel Rocha
Alguém poderia, por favor, solucionar o problema abaixo:

Ache a solução real da equação:
log[(sqrt 2)^(x-2)] = x

-- 
Esta mensagem foi verificada pelo sistema de antiv�rus e
 acredita-se estar livre de perigo.



[obm-l] Re: [obm-l] Re: [obm-l] Re: [obm-l] Dúvida em Geometria Plana

2016-06-05 Por tôpico Carlos Gomes
De nada amigo! Sempre um prazer qdo posso ajudar!

Abraço, Cgomes.

Em 2 de junho de 2016 19:03, Daniel Rocha 
escreveu:

> Muito Obrigado, Carlos !!!
>
> Em 2 de junho de 2016 18:54, Carlos Gomes  escreveu:
>
>> Seja x a medida do ângulo BAC. Como o triângulo APQ é isosceles de base
>> AP, segue q a medida do ângulo APQ também é x. Note que o ângulo BQP é
>> externo ao triângulo APQ, portanto, mede x+x=2x. Agora como o triângulo BQP
>> é isosceles de base BQ, segue que o ângulo PBQ também mede 2x. Por fim note
>> que o ângulo BPC é externo ao triângulo  ABP, portanto mede x+2x=3x...como
>> o triângulo BCP também é isosceles de base PC, segue que o ângulo PCB
>> também mede 3x...como o triângulo ABC é isosceles, segue que o ângulo ABC
>> também mede 3x, o que revela q o ângulo PCB mede x. Assim, no triângulo BCP
>> temos que
>> x+3x+3x=π   ==>x=π/7.
>> Em 2 de jun de 2016 18:32, "Daniel Rocha" 
>> escreveu:
>>
>>> Olá a todos,
>>>
>>> Alguém poderia, por favor, apresentar os cálculos corretos da seguinte
>>> questão:
>>>
>>> Considere um triângulo ABC isósceles de base BC, e os pontos P e Q tais
>>> que P pertence a AC e Q pertence a AB. Se BC=BP=PQ=QA, a medida do ângulo
>>> do vértice A, em radianos, é:
>>>
>>> GABARITO: Pi/7.
>>>
>>> Eu agradeço a quem apresentar os cálculos corretos.
>>>
>>> --
>>> Esta mensagem foi verificada pelo sistema de antivírus e
>>> acredita-se estar livre de perigo.
>>
>>
>> --
>> Esta mensagem foi verificada pelo sistema de antivírus e
>> acredita-se estar livre de perigo.
>
>
>
> --
> Esta mensagem foi verificada pelo sistema de antivírus e
> acredita-se estar livre de perigo.
>

-- 
Esta mensagem foi verificada pelo sistema de antiv�rus e
 acredita-se estar livre de perigo.



[obm-l] Re: [obm-l] Re: [obm-l] Dúvida em Geometria Plana

2016-06-02 Por tôpico Daniel Rocha
Muito Obrigado, Carlos !!!

Em 2 de junho de 2016 18:54, Carlos Gomes  escreveu:

> Seja x a medida do ângulo BAC. Como o triângulo APQ é isosceles de base
> AP, segue q a medida do ângulo APQ também é x. Note que o ângulo BQP é
> externo ao triângulo APQ, portanto, mede x+x=2x. Agora como o triângulo BQP
> é isosceles de base BQ, segue que o ângulo PBQ também mede 2x. Por fim note
> que o ângulo BPC é externo ao triângulo  ABP, portanto mede x+2x=3x...como
> o triângulo BCP também é isosceles de base PC, segue que o ângulo PCB
> também mede 3x...como o triângulo ABC é isosceles, segue que o ângulo ABC
> também mede 3x, o que revela q o ângulo PCB mede x. Assim, no triângulo BCP
> temos que
> x+3x+3x=π   ==>x=π/7.
> Em 2 de jun de 2016 18:32, "Daniel Rocha" 
> escreveu:
>
>> Olá a todos,
>>
>> Alguém poderia, por favor, apresentar os cálculos corretos da seguinte
>> questão:
>>
>> Considere um triângulo ABC isósceles de base BC, e os pontos P e Q tais
>> que P pertence a AC e Q pertence a AB. Se BC=BP=PQ=QA, a medida do ângulo
>> do vértice A, em radianos, é:
>>
>> GABARITO: Pi/7.
>>
>> Eu agradeço a quem apresentar os cálculos corretos.
>>
>> --
>> Esta mensagem foi verificada pelo sistema de antivírus e
>> acredita-se estar livre de perigo.
>
>
> --
> Esta mensagem foi verificada pelo sistema de antivírus e
> acredita-se estar livre de perigo.

-- 
Esta mensagem foi verificada pelo sistema de antiv�rus e
 acredita-se estar livre de perigo.



[obm-l] Re: [obm-l] Dúvida em Geometria Plana

2016-06-02 Por tôpico Carlos Gomes
Seja x a medida do ângulo BAC. Como o triângulo APQ é isosceles de base AP,
segue q a medida do ângulo APQ também é x. Note que o ângulo BQP é externo
ao triângulo APQ, portanto, mede x+x=2x. Agora como o triângulo BQP é
isosceles de base BQ, segue que o ângulo PBQ também mede 2x. Por fim note
que o ângulo BPC é externo ao triângulo  ABP, portanto mede x+2x=3x...como
o triângulo BCP também é isosceles de base PC, segue que o ângulo PCB
também mede 3x...como o triângulo ABC é isosceles, segue que o ângulo ABC
também mede 3x, o que revela q o ângulo PCB mede x. Assim, no triângulo BCP
temos que
x+3x+3x=π   ==>x=π/7.
Em 2 de jun de 2016 18:32, "Daniel Rocha" 
escreveu:

> Olá a todos,
>
> Alguém poderia, por favor, apresentar os cálculos corretos da seguinte
> questão:
>
> Considere um triângulo ABC isósceles de base BC, e os pontos P e Q tais
> que P pertence a AC e Q pertence a AB. Se BC=BP=PQ=QA, a medida do ângulo
> do vértice A, em radianos, é:
>
> GABARITO: Pi/7.
>
> Eu agradeço a quem apresentar os cálculos corretos.
>
> --
> Esta mensagem foi verificada pelo sistema de antivírus e
> acredita-se estar livre de perigo.

-- 
Esta mensagem foi verificada pelo sistema de antiv�rus e
 acredita-se estar livre de perigo.



[obm-l] Dúvida em Geometria Plana

2016-06-02 Por tôpico Daniel Rocha
Olá a todos,

Alguém poderia, por favor, apresentar os cálculos corretos da seguinte
questão:

Considere um triângulo ABC isósceles de base BC, e os pontos P e Q tais que
P pertence a AC e Q pertence a AB. Se BC=BP=PQ=QA, a medida do ângulo do
vértice A, em radianos, é:

GABARITO: Pi/7.

Eu agradeço a quem apresentar os cálculos corretos.

-- 
Esta mensagem foi verificada pelo sistema de antiv�rus e
 acredita-se estar livre de perigo.



[obm-l] Re: [obm-l] Dúvida função

2016-01-30 Por tôpico Esdras Muniz
Teorema: Se f: R ---> Y é contínua e X é compacto, então f admite um máximo
e um mínimo em X.

Em 28 de janeiro de 2016 23:16, Israel Meireles Chrisostomo <
israelmchrisost...@gmail.com> escreveu:

> Olá pessoal eu gostaria de provar que uma função admite máximo sem
> calcular o máximo da função, isto é possível?
> Por exemplo, seja f(a,b,c) uma função, eu quero provar que a,b,c admite
> máximo sem calcular seu máximo, lembrando f(a,b,c) é uma função de 3
> variáveis, alguém por favor poderia me ajudar?
>



-- 
Esdras Muniz Mota
Mestrando em Matemática
Universidade Federal do Ceará


[obm-l] Dúvida função

2016-01-28 Por tôpico Israel Meireles Chrisostomo
Olá pessoal eu gostaria de provar que uma função admite máximo sem calcular
o máximo da função, isto é possível?
Por exemplo, seja f(a,b,c) uma função, eu quero provar que a,b,c admite
máximo sem calcular seu máximo, lembrando f(a,b,c) é uma função de 3
variáveis, alguém por favor poderia me ajudar?


[obm-l] Dúvida aparentemente simples

2015-08-11 Por tôpico Vanderlei Nemitz
Trabalho com edição de material didático e outro dia, em conversa com um
autor de material do ensino médio, surgiu uma questão sobre a qual gostaria
de saber a opinião de vocês.

O oposto de zero é zero ou não faz sentido falar dele?

Por um lado, a solução da equação x = -x é x = 0, ou seja, poderíamos dizer
que -0 = 0 (o oposto de zero é zero).

Por outro, trata-se de um único número (ele e seu oposto) e assim não faz
sentido falar dele.

A pergunta foi motivada por outra:

Para quais valores de x temos que sqrt(x^2) = -x?

A resposta é x  0 ou x= 0?


Obrigado!

Vanderlei

-- 
Esta mensagem foi verificada pelo sistema de antiv�rus e
 acredita-se estar livre de perigo.



[obm-l] Re: [obm-l] Dúvida aparentemente simples

2015-08-11 Por tôpico Ralph Teixeira
Acho que a convencao de quase todos eh que -0=0. Nao vejo problema de que o
oposto de algo seja ele mesmo.

2015-08-11 11:27 GMT-03:00 Vanderlei Nemitz vanderma...@gmail.com:

 Trabalho com edição de material didático e outro dia, em conversa com um
 autor de material do ensino médio, surgiu uma questão sobre a qual gostaria
 de saber a opinião de vocês.

 O oposto de zero é zero ou não faz sentido falar dele?

 Por um lado, a solução da equação x = -x é x = 0, ou seja, poderíamos
 dizer que -0 = 0 (o oposto de zero é zero).

 Por outro, trata-se de um único número (ele e seu oposto) e assim não faz
 sentido falar dele.

 A pergunta foi motivada por outra:

 Para quais valores de x temos que sqrt(x^2) = -x?

 A resposta é x  0 ou x= 0?


 Obrigado!

 Vanderlei

 --
 Esta mensagem foi verificada pelo sistema de antivírus e
 acredita-se estar livre de perigo.

-- 
Esta mensagem foi verificada pelo sistema de antiv�rus e
 acredita-se estar livre de perigo.



[obm-l] Re: [obm-l] Re: [obm-l] Dúvida aparentemente simples

2015-08-11 Por tôpico Pedro José
Boa tarde!

Pela definição, simétrico ou oposto de um elemento a de um anel é o
elemento do anel que operado com a por + resulte 0.
Portanto o simétrico ou oposto de zero é zero.

Saudações,
PJMS.

Em 11 de agosto de 2015 12:02, Ralph Teixeira ralp...@gmail.com escreveu:

 Acho que a convencao de quase todos eh que -0=0. Nao vejo problema de que
 o oposto de algo seja ele mesmo.

 2015-08-11 11:27 GMT-03:00 Vanderlei Nemitz vanderma...@gmail.com:

 Trabalho com edição de material didático e outro dia, em conversa com um
 autor de material do ensino médio, surgiu uma questão sobre a qual gostaria
 de saber a opinião de vocês.

 O oposto de zero é zero ou não faz sentido falar dele?

 Por um lado, a solução da equação x = -x é x = 0, ou seja, poderíamos
 dizer que -0 = 0 (o oposto de zero é zero).

 Por outro, trata-se de um único número (ele e seu oposto) e assim não faz
 sentido falar dele.

 A pergunta foi motivada por outra:

 Para quais valores de x temos que sqrt(x^2) = -x?

 A resposta é x  0 ou x= 0?


 Obrigado!

 Vanderlei

 --
 Esta mensagem foi verificada pelo sistema de antivírus e
 acredita-se estar livre de perigo.



 --
 Esta mensagem foi verificada pelo sistema de antivírus e
 acredita-se estar livre de perigo.


-- 
Esta mensagem foi verificada pelo sistema de antiv�rus e
 acredita-se estar livre de perigo.



[obm-l] Re: [obm-l] Re: [obm-l] dúvida

2015-07-09 Por tôpico Ralph Teixeira
Vamos generalizar para R^n: com a noção usual (Euclideana) de comprimento,
o comprimento do segmento que liga (x1,x2,...,xn) a (y1,y2,...,yn) é:

d=raiz((y1-x1)^2+(y2-x2)^2+...+(yn-xn)^2)

Esta é a noção usual de distância entre dois pontos -- confira que é o que
você conhece na reta (n=1) e no plano (n=2).

Abraço, Ralph.

2015-07-09 10:27 GMT-03:00 Pedro José petroc...@gmail.com:

 Bom dia!

 E o segmento???

 Em 8 de julho de 2015 21:48, Israel Meireles Chrisostomo 
 israelmchrisost...@gmail.com escreveu:

 Como posso encontrar o comprimento de um segmento de reta no espaço
 tridimensional?Considere a origem da reta no ponto (x_0,y_0,z_0) e o final
 da reta no ponto (x_1,y_1,z_1)

 --
 Esta mensagem foi verificada pelo sistema de antivírus e
 acredita-se estar livre de perigo.



 --
 Esta mensagem foi verificada pelo sistema de antivírus e
 acredita-se estar livre de perigo.

-- 
Esta mensagem foi verificada pelo sistema de antiv�rus e
 acredita-se estar livre de perigo.



[obm-l] Re: [obm-l] dúvida

2015-07-09 Por tôpico Pedro José
Bom dia!

E o segmento???

Em 8 de julho de 2015 21:48, Israel Meireles Chrisostomo 
israelmchrisost...@gmail.com escreveu:

 Como posso encontrar o comprimento de um segmento de reta no espaço
 tridimensional?Considere a origem da reta no ponto (x_0,y_0,z_0) e o final
 da reta no ponto (x_1,y_1,z_1)

 --
 Esta mensagem foi verificada pelo sistema de antivírus e
 acredita-se estar livre de perigo.

-- 
Esta mensagem foi verificada pelo sistema de antiv�rus e
 acredita-se estar livre de perigo.



[obm-l] Re: [obm-l] Re: [obm-l] Re: [obm-l] dúvida

2015-07-09 Por tôpico Israel Meireles Chrisostomo
Obrigado Ralph

Em 9 de julho de 2015 12:37, Ralph Teixeira ralp...@gmail.com escreveu:

 Vamos generalizar para R^n: com a noção usual (Euclideana) de comprimento,
 o comprimento do segmento que liga (x1,x2,...,xn) a (y1,y2,...,yn) é:

 d=raiz((y1-x1)^2+(y2-x2)^2+...+(yn-xn)^2)

 Esta é a noção usual de distância entre dois pontos -- confira que é o que
 você conhece na reta (n=1) e no plano (n=2).

 Abraço, Ralph.

 2015-07-09 10:27 GMT-03:00 Pedro José petroc...@gmail.com:

 Bom dia!

 E o segmento???

 Em 8 de julho de 2015 21:48, Israel Meireles Chrisostomo 
 israelmchrisost...@gmail.com escreveu:

 Como posso encontrar o comprimento de um segmento de reta no espaço
 tridimensional?Considere a origem da reta no ponto (x_0,y_0,z_0) e o final
 da reta no ponto (x_1,y_1,z_1)

 --
 Esta mensagem foi verificada pelo sistema de antivírus e
 acredita-se estar livre de perigo.



 --
 Esta mensagem foi verificada pelo sistema de antivírus e
 acredita-se estar livre de perigo.



 --
 Esta mensagem foi verificada pelo sistema de antivírus e
 acredita-se estar livre de perigo.


-- 
Esta mensagem foi verificada pelo sistema de antiv�rus e
 acredita-se estar livre de perigo.



[obm-l] Dúvida sistemas lineares

2015-07-08 Por tôpico Israel Meireles Chrisostomo
Dados a,b e c reais quaisquer sempre existem x,y e z tais que a=y+z, b=x+z
e c=x+y?

-- 
Esta mensagem foi verificada pelo sistema de antiv�rus e
 acredita-se estar livre de perigo.



[obm-l] dúvida

2015-07-08 Por tôpico Israel Meireles Chrisostomo
Como posso encontrar o comprimento de um segmento de reta no espaço
tridimensional?Considere a origem da reta no ponto (x_0,y_0,z_0) e o final
da reta no ponto (x_1,y_1,z_1)

-- 
Esta mensagem foi verificada pelo sistema de antiv�rus e
 acredita-se estar livre de perigo.



  1   2   3   4   5   6   7   8   9   10   >